Krok 2 - 2010 Question Paper (Stomatology)

28
Krok 2 Stomatology 2010 1 1. A 42-year-old patient complai- ns about general weakness. Body temperature is 41 o C , he presents with si- ckness, emesis, dyspnea. There is evident well-dened skin redness in form of ame tips on her face. Submandibular lymph nodes are enlarged. What is the most li- kely diagnosis? A. Erysipelas B. Streptococcal impetigo C. Phlebitis of facial vein D. Lupus E. Anthrax 2. A victim got a perforating wound of the left cheek contaminated with radioactive materials on the battleeld. The wound was dressed with aseptic bandage from the individual rst-aid pack and processed with anaesthetic and antibacterial medicaments. The dressing should be changed at the following stati- on of medical evacuation: A. At each station of evacuation B. At the battalion aid station C. At the regimental aid station D. At the station of the separate medical battalion E. Only when the dressing is unsatisfactory 3. Mother of a 6,5-year-old child consulted a dentist about checking ssure hermeti- zatioon of the 16, 26, 36, 46 teeth. They were treated six month ago. Sealant in the 36, 46 teeth is preserved, but sealant in the 16 and 26 teeth is absent. What is the tactics of choice? A. To repeat ssure hermetization B. Preventive lling C. Applications with remodentum solution D. Coating the teeth with uorine lacquer Ftorlak E. Electrophoresis with 1% sodium uori- de solution 4. A 12-year-old boy complains about short-term pain in a lower jaw tooth on the right caused by cold stimuli. Objecti- vely: the 46 tooth has a carious cavi- ty on the masticatory surface within the circumpulpar dentine which has no intercommunication with the dental cavi- ty. The cavity oor and walls are coated with light softened dentine. Cold test causes short-standing pain. What is the most likely diagnosis? A. Acute deep caries B. Acute median caries C. Acute localized pulpitis D. Acute diffuse pulpitis E. Chronic brous pulpitis 5. A 12-year-old child complains about sore throat, headache, body temperature rise up to 38, 5 o C , rhinitis, cough in summer period. Objectively: mucous membrane of oral cavity is hyperemic, edematic. There are 10-15 erosions up to 0,5 mm large on the palate and palatine arches, that aren’t covered with deposit and have red oor. Regional lymph nodes are enlarged and painful on palpation. What is the most likely diagnosis? A. Herpetic angina B. Acute herpetic stomatitis C. Erythema multiforme D. Chronic recurrent aphthous stomatitis E. Infectious mononucleosis 6. A 43-year-old man complains about pai- nful swallowing, limited mouth opening. Destroyed 37 tooth caused pain 3 days ago. Painful swallowing appeared the next day. Limited mouth opening was present a day later. Objectively: there is a small soft swelling in the left submaxillary area. An enlarged lymph node is palpated. The mouth can be opened by 1,5 cm. Edema of left palatine arch and pharynx lateral wall is present. The 37 tooth is destroyed. Mucous membrane around the tooth is edematic. Percussion is slightly painful. What is the most likely diagnosis? A. Odontogenic peripharyngeal phlegmon B. Odontogenic pterygomandibular phlegmon C. Odontogenic submaxillary phlegmon D. Peritonsillar abscess E. Acute submaxillary lymphadenitis 7. A 34-year-old male patient consults a dentist about an ulcer on the hard palate. It appeared about a month ago. He has treated it by rinsing with herbal water, but the ulcer is gradually "creeping". Objecti- vely: there is a shallow erethistic ulcer wi- th uneven and undermined edges of soft consistency within the mucous membrane of hard palate. Granulations of the ulcer oor are also present. Yellowish granules are visible on the ulcer periphery. What is the most likely diagnosis?

Transcript of Krok 2 - 2010 Question Paper (Stomatology)

Page 1: Krok 2 - 2010 Question Paper (Stomatology)

Krok 2 Stomatology 2010 1

1. A 42-year-old patient complai-ns about general weakness. Bodytemperature is 41oC, he presents with si-ckness, emesis, dyspnea. There is evidentwell-defined skin redness in form of flametips on her face. Submandibular lymphnodes are enlarged. What is the most li-kely diagnosis?

A. ErysipelasB. Streptococcal impetigoC. Phlebitis of facial veinD. LupusE. Anthrax

2. A victim got a perforating woundof the left cheek contaminated withradioactive materials on the battlefield.The wound was dressed with asepticbandage from the individual first-aidpack and processed with anaesthetic andantibacterial medicaments. The dressingshould be changed at the following stati-on of medical evacuation:

A. At each station of evacuationB. At the battalion aid stationC. At the regimental aid stationD. At the station of the separate medicalbattalionE. Only when the dressing is unsatisfactory

3. Mother of a 6,5-year-old child consulteda dentist about checking fissure hermeti-zatioon of the 16, 26, 36, 46 teeth. Theywere treated six month ago. Sealant in the36, 46 teeth is preserved, but sealant inthe 16 and 26 teeth is absent. What is thetactics of choice?

A. To repeat fissure hermetizationB. Preventive fillingC. Applications with remodentum solutionD. Coating the teeth with fluorine lacquerFtorlakE. Electrophoresis with 1% sodium fluori-de solution

4. A 12-year-old boy complains aboutshort-term pain in a lower jaw tooth onthe right caused by cold stimuli. Objecti-vely: the 46 tooth has a carious cavi-ty on the masticatory surface withinthe circumpulpar dentine which has nointercommunication with the dental cavi-ty. The cavity floor and walls are coatedwith light softened dentine. Cold testcauses short-standing pain. What is themost likely diagnosis?

A. Acute deep cariesB. Acute median cariesC. Acute localized pulpitisD. Acute diffuse pulpitisE. Chronic fibrous pulpitis

5. A 12-year-old child complains aboutsore throat, headache, body temperaturerise up to 38, 5oC, rhinitis, cough insummer period. Objectively: mucousmembrane of oral cavity is hyperemic,edematic. There are 10-15 erosions up to0,5 mm large on the palate and palatinearches, that aren’t covered with depositand have red floor. Regional lymph nodesare enlarged and painful on palpation.What is the most likely diagnosis?

A. Herpetic anginaB. Acute herpetic stomatitisC. Erythema multiformeD. Chronic recurrent aphthous stomatitisE. Infectious mononucleosis

6. A 43-year-old man complains about pai-nful swallowing, limited mouth opening.Destroyed 37 tooth caused pain 3 daysago. Painful swallowing appeared the nextday. Limited mouth opening was presenta day later. Objectively: there is a smallsoft swelling in the left submaxillary area.An enlarged lymph node is palpated. Themouth can be opened by 1,5 cm. Edemaof left palatine arch and pharynx lateralwall is present. The 37 tooth is destroyed.Mucous membrane around the tooth isedematic. Percussion is slightly painful.What is the most likely diagnosis?

A. Odontogenic peripharyngeal phlegmonB. Odontogenic pterygomandibularphlegmonC. Odontogenic submaxillary phlegmonD. Peritonsillar abscessE. Acute submaxillary lymphadenitis

7. A 34-year-old male patient consults adentist about an ulcer on the hard palate.It appeared about a month ago. He hastreated it by rinsing with herbal water, butthe ulcer is gradually "creeping". Objecti-vely: there is a shallow erethistic ulcer wi-th uneven and undermined edges of softconsistency within the mucous membraneof hard palate. Granulations of the ulcerfloor are also present. Yellowish granulesare visible on the ulcer periphery. What isthe most likely diagnosis?

Page 2: Krok 2 - 2010 Question Paper (Stomatology)

Krok 2 Stomatology 2010 2

A. Tuberculous ulcerB. Trophic ulcerC. Cancerous ulcerD. Syphilitic ulcerE. Actinomycosis

8. A 27-year-old male patient underwentextraction of the medial root of the 36tooth, but the distal tooth root was brokenin its middle third. What tools should beapplied for root extraction?

A. Left angled elevatorB. Right angled elevatorC. Straight elevatorD. Broad-beaked forcepsE. Close-beaked forceps

9. A 55-year-old woman complains aboutliquid pouring through her nose duringeating of fluid food. Objectively: there isa perforating defect of alveolar process3x2,8 cm deep in the lateral part of theupper jaw at level with the extracted16 tooth. The patient refused operativeintervention. What construction shouldbe offered?

A. Minor saddle prosthesis with claspfixationB. Clasp dental prosthesis with obturatingelement made of elastic plasticC. Partial removable lamellar prosthesisD. Fixed dental bridgeE. Celluloid mouth protector

10. A 53-year-old patient was diagnosedwith leucoplakia nicotinica Tappeiner.What pathohistological process predomi-nates in histologic pattern of this disease?

A. HyperkeratosisB. AcanthosisC. DyskeratosisD. ParakeratosisE. Papillomatosis

11. A 24-year-old female patient complai-ns about hard tissues defect of the21 tooth. Objectively: the 21 tooth isdestroyed by 1/3 along the cutting edge,it is changed in color. X-ray picture showsthat the root canal is filled to the top. Itwas decided to fabricate a plastic crown.Where should the crown edge be situatedagainst the marginal gingiva?

A. At a level with gingivaB. 0,5 mm away of gingivaC. 1,0 mm away of gingivaD. 0,5 mm below the gingivaE. 1,0 mm below the gingiva

12. A 70-year-old male patient orderedcomplete removable dentures for bothjaws. In the clinical stage he came for"identification of central relationship".For verification of interalveolar height thepatient underwent speaking test. The pati-ent was asked to pronounce the sounds"O", "E", "M". What distance shouldbe between the occlusal rims for correctidentification of interalveolar height?

A. 5-6 mmB. 9-10 mmC. 3-4 mmD. 1-2 mmE. 7-8 mm

13. A 16-year-old male patient complainsabout pain in the oral cavity, ulceration,body temperatureup to 38oC, headache.Objectively: mucous membrane of theoral cavity is hyperemic and edematic.There are a lot of confluent erosions ofpolycyclic shape, covered with grey andwhite deposit, located on hard palate,gums, lips. What is your provisional di-agnosis?

A. Acute herpetic stomatitisB. Erythema multiformeC. Pemphigus vulgarisD. Aphthous feverE. Allergic stomatitis

14. An 8-year-old child was diagnosedwith granulating periodontitis of the 55tooth. The crown is completely destroyed.X-ray picture shows separated tooth roots.Choose an optimal tool for operativeintervention:

A. Root bayonet-shaped forcepsB. Root straight forcepsC. Root beak-shaped forcepsD. Crown bayonet-shaped forcepsE. Crown S-shaped forceps

15. An 8-year-old boy was diagnosed withchronic fibrous pulpitis of the 21 tooth.It was treated by extirpation method.Choose the material for root filling:

A. Calcium-containing materialB. Glass-ionomer cementC. Zinc oxide eugenol pasteD. Resorcin-formalin pasteE. Phosphate cement

16. A 12-year-old female patient was di-agnosed with open bite and dentoalveolarelongation of lateral part of mandible.What construction of apparatus is requi-red?

Page 3: Krok 2 - 2010 Question Paper (Stomatology)

Krok 2 Stomatology 2010 3

A. Upper jaw appliance with occlusal restseatsB. Extraoral face bowC. Angle’s sliding face bowD. Herbst applianceE. Upper jaw appliance with a face bow

17. An 18-year-old patient complainsabout an aesthetic defect. Objectively: thelower teeth are set forward and overlapthe upper antagonists. This symptom istypical for the following bite abnormality:

A. Mesial biteB. Distal biteC. Deep biteD. Open biteE. Cross bite

18. A 70-year-old male patient complainsabout total teeth missing on both jaws.It is planned to fabricate a completeremovable lower jaw prosthesis. Objecti-vely: alveolar process of the lower jawis atrophied only in the frontal aspect.Identify the type of lower jaw atrophyaccording to Keller’s classification:

A. IVB. IIC. ID. IIIE. -

19. A 18-year-old female patient ordered ametal-ceramic crown for the 24 tooth. It isnecessary to choose an optimal impressi-on material for combined impression ofthe upper jaw. What material should beused for a preliminary impression?

A. StensB. DentafolC. OrthocarD. WaxE. Protacryl

20. A patient consulted a dentist aboutconstant dull pain in the 38 tooth. Mouthopening is limited down to 1 cm. It isnecessary to extract the 38 tooth. Whatkind of anesthesia should be applied foroperation?

A. Bercher-DubovB. MandibularC. TorusD. Extraoral mandibular techniqueE. Tuberal

21. One minute after perfoming torusanaesthesia with 2% novocaine solution

(4ml) during extraction of the 17 tooththe patient complained about respirationobstruction. Objectively: upper and lowerlips are edematic, mucous membrane oflarynx and oral cavity is edematic andextremely hyperemic. What complicationturned up in this patient?

A. Quincke’s edemaB. Anesthetic intoxicationC. Anaphylactic shockD. CollapseE. Acute pulmonary insufficiency

22. A 37-year-old male patient complai-ns about pain of the 46 tooth duringfood intake, especially hot food, offensivebreath when he sucks his tooth. Objecti-vely: the face is symmetrical, masticatorysurface of the 48 tooth has a deep cari-ous cavity communicating with the dentalcavity. X-ray picture shows widening ofperiodontal fissure at the root apex ofthe 46 tooth. What is the most likely di-agnosis?

A. Chronic gangrenous pulpitisB. Exacerbation of chronic periodontitisC. Exacerbation of chronic pulpitisD. Chronic fibrous periodontitisE. Chronic fibrous pulpitis

23. A 17-year-old girl complains about pai-nfulness and gingival hemorrhage duringtooth brushing and food intake. She hasbeen suffering from it for 1,5 year. Sheconsulted a dentist. There was no consi-derable effect from treatment. Objecti-vely: gingival papillae are flabby andcyanotic in the region of frontal teethon both jaws, they overlay teeth crownsby 1/3-1/2 of their height, they bleed ontouch. Therapeutic effect can be achievedby means of electrophoresis with:

A. HeparinB. AloeC. Vitamin B1

D. TripsinE. Sodium fluoride

24. A 38-year-old female patient complai-ns about pain in projection of the externalacoustic meatus, clicking during mouthopening, ear stuffiness. Objectively: theface is symmetric, mouth is opened instaright path. There is Kennedy I typedentition defect, the 18, 17, 16, 26, 27,28 teeth are missing. What anatomicalformation carries the maximum load?

Page 4: Krok 2 - 2010 Question Paper (Stomatology)

Krok 2 Stomatology 2010 4

A. Articular disk (meniscus)B. Articular headC. Distal clivus of medial articular tubercleD. Glenoid fossa floor of temporal boneE. Joint capsule

25. Physical examination of a patientrevealed a hard tissues defect of the 24tooth. Index of occlusal surface destructi-on was 0,8. The defect was restored byfilling that doesn’t meet the requirements.The tooth is of rose colouring. X-raypicture shows no patological processes.What construction is indicated in thiscase?

A. Pivot crownB. Artificial crownC. InlayD. Portion crownE. Equator crown

26. A 49-year-old male patient consulteda dental surgeon about an enlarging"mother’s mark", itch and desquamati-on. According to patient, changed in colorand size skin area appeared a year agoafter a shaving trauma. Objectively: thereis an intensely pigmented brown spot upto 2 cm large with small oval protrudingnodules on its surface in the infraorbitalregion on the left. Desquamation factorsare present. Palpation is painless. Regi-onal lymph nodes are matted together wi-th skin and painless. What is the most li-kely diagnosis?

A. MelanomaB. Pigmented nevusC. Verrucous nevusD. Epidermoid cancerE. Papillomatosis

27. A soldier was delivered to the regimentmedical station with dislocation asphyxiacaused by a gunshot wound. What actionsshould be taken for asphyxia suppression?

A. Pul the tongue forwards and sew itthroughB. Intubation of tracheaC. Tracheostoma establishmentD. Artificial airwayE. Artificial pulmonary ventilation

28. A 58-year-old male patient has consi-derable microstomia as a result of aface trauma and burn. The patient hasindication for fabrication of a partialremovable prosthesis. What constructionof prosthesis should be used in this case?

A. Collapsible prosthesisB. Clasp dental prosthesisC. Clasp dental prosthesis withattachmentsD. Pivot-point removable prosthesisE. Partial removable lamellar prosthesis

29. X-ray picture depicts a circular well-defined area of bone tissue destruction0,7х 0,7 cm large in the projection of rootapex. What is the most likely diagnosis?

A. CystogranulomaB. CystC. GranulomaD. OdontomaE. Osteoma

30. A 32-year-old patient complainsabout a cosmetic defect caused by parti-al teeth missing. Objectively: the 15,14, 24, 25 teeth are missing, the centralgroup of upper jaw teeth is in protrusi-on and presents with diasthemas. Whatorthodontic appliance should be used toprepare the oral cavity for prosthetics?

A. Palatine plate with vestibular bowB. Schwarz’ guardC. Palatine plate with oblique plane andvestibular bowD. Brueckl’s applianceE. Expansion plate for the upper jaw withorthodontic lock

31. A 35-year-old patient has teeth mobi-lity of I degree, the teeth have apparentdental cusps. The dentist recommends todo occlusive teeth grinding. What methodis the most objective for determining thegrinding topography?

A. OcclusiographyB. MasticatiographyC. RoentgenographyD. GnathodynamometryE. Rubinov’s masticatory test

32. A 3-year-old girl complains aboutpain and tumescence in the region of thedecayed 51, 52 teeth, body temperaturerise up to 37, 5 − 37, 9oC. Objectively:the face is asymmetric because of atumescence in the upper lip region andright infraorbital region. The crown of the51 tooth is completely decayed. Mucousmembrane in the region of the 52, 51,1 teeth is edematic, mucogingival fold issmoothed, palpation provokes pain, mobi-lity of I-II degree of the 51, 52 teeth is alsopresent. What is the most likely diagnosis?

Page 5: Krok 2 - 2010 Question Paper (Stomatology)

Krok 2 Stomatology 2010 5

A. Acute purulent odontogenic maxillaryperiostitisB. Acute albuminous odontogenic maxi-llary periostitisC. Acute odontogenic maxillaryosteomyelitisD. Odontogenic abscess of infraorbitalregionE. Exacerbation of chronic periodontitisof the 51 tooth

33. A 6-year-old boy hit his forehead oneday ago. A few hours later a swellingappeared in the right superciliary regi-on. Objectively: there is a considerableedema of forehead tissues spreading toeyelids of the right eye, the skin over theswelling is cyanotic, the swelling is of softconsistency. Fluctuation is also present.General condition of the boy is normal.Make a provisional diagnosis:

A. Hematoma of the right superciliaryregionB. Postraumatic edema of tissues of theright superciliary regionC. Fracture of frontal boneD. Hematic abscess of the right supercili-ary regionE. Inflammatory infiltration of tissues ofthe right superciliary region

34. A 45-year-old patient complains aboutpain in his mandible that arose afterextraction of the 36 tooth. Objectively:alveolar socket is covered with bloodyclot. X-ray picture shows unextracted rootof the 36 tooth. What tools are necessaryfor extraction of this root?

A. Angled elevator curved rightB. Angled elevator curved leftC. Straight elevatorD. S-shaped forcepsE. Bayonet-shaped forceps

35. A 70-year-old patient consulted adental surgeon about extraction of thecentral upper jaw incisors with III degreemobility. What tools should be applied?

A. Straight forcepsB. Straight elevatorC. Beak-shaped forcepsD. Bayonet-shaped forcepsE. S-shaped forceps

36. A patient complains of pain and swelli-ng in the right submandibular area. Shehas been treating the 45 tooth for a week.Objectively: body temperature is 38oC.There is a painful tense infiltration inthe right submandibular region. The skin

doesn’t make a fold, its hyperemic andglossy. The mouth can be opened by 3 cm.Deglutition is painless. These clinical fi-ndings correspond with the following di-sease:

A. Odontogenous phlegmon of the rightsubmandibular regionB. Abscess of the right alveololingualgrooveC. Adenophlegmon of the right submandi-bular regionD. Acute odontogenous sialoadenitisE. Phlegmon of pterygomandibular space

37. Preventive examination of tongueback of a 6-year-old child revealed areasof epithelium desquamation in form ofred oval spots located close to the zonesof hyperkeratinization of filiform papillae.Clavate papillae are hypertrophic. Thereare no complaints. The child has a historyof intestinal dysbacteriosis. What is themost likely diagnosis?

A. Glossitis areata exfoliativaB. Candidal glossitisC. Acute catarrhal glossitisD. Rhomboid glossitisE. Herpetic affection of tongue

38. During endodontic treatment of peri-odontitis a tool was broken in the mi-ddle third of the medial root. Choose thetreatment method:

A. Medial root hemisectionB. Medial root amputationC. Crown radicular separationD. Resection of medial root apexE. Tooth extraction

39. A 14-year-old child complains about acosmetic defect in the frontal teeth regi-on. Objectively: enamel of the 11, 12,21, 22, 31, 32, 41, 42 teeth is thin in theregion of cutting edge, there is a sulcateenamel pit 1,5 mm wide which encirclesthe tooth and is parallel to the cuttingedge. The cusps of the 16, 26, 36, 46 teethare underdeveloped and have conicalform. What is the most likely diagnosis?

A. Systemic hypoplasiaB. Dentinogenesis imperfectaC. Enamel dysplasiaD. Local hypoplasiaE. Stainton-Capdepont dysplasia

40. Examination of a 9-year-old childrevealed protrudent chin, the lower lipoverlapping the upper lip. There are di-astemas and tremas beetwen the lower

Page 6: Krok 2 - 2010 Question Paper (Stomatology)

Krok 2 Stomatology 2010 6

incisors, the lower incisors overlap theupper ones by 2/3 of crown height. Sagi-ttal fissure is 3 mm. Specify the treatmenttactics:

A. Brueckl’s applianceB. Bynin’s guardC. Schwartz’ quardD. Angle’s sliding applianceE. Myogymnastics complex

41. It is planned to organize a dental roomwith three universal dental sets. The totalarea of the room must be:

A. No less than 28 square metresB. No less than 21 square metresC. No less than 30 square metresD. No less than 42 square metresE. No less than 18 square metres

42. During pulpitis treatment of the 25tooth a 30-year-old patient received aninjection of 2% lidocaine as anaesthetic.A few minutes after the injection thpatient presented with numb tongue,asphyxia, dry cough, cyanosis, anxiety,convulsions. Arterial pressure fell downto 90/60 mm Hg. What is the most likelydiagnosis?

A. Anaphylactic shockB. Acute heart failureC. SyncopeD. Quincke’s edemaE. Epilepsy

43. A 50-year-old patient has medianlower jaw fracture with formation of falsejoint. The 38, 32, 31, 41, 42, 48 teeth aremissing. The remaining teeth are intact,stable. There is no displacement of lowerjaw fragments. X-ray picture shows a bonetissue defect 1 cm large. What prosthesisis indicated?

A. Oxman’s bridge-like prosthesis withpivot pointB. Clasp dentureC. Lamellar prosthesis with Gavrilow’spivot pointD. Lamellar prosthesis with Oxman’s pivotpointE. Bridge-like prosthesis without a pivotpoint

44. A 60-year-old male patient has anextensive defect of nose caused by aninjury. The patient temporarily refusedoperative intervention, it was suggestedto fabricate an external nasal prosthesisfixed by eyeglass frame. What impressionmaterial should be used?

A. Gypseous impression of the whole faceB. Impression of the whole face withthermoplastic materialC. Impression of the defect with elasticmaterialD. Impression of the defect withthermoplastic massE. Gypseous impression

45. A 53-year-old patient consulted aprosthodontist about lateral teeth mobili-ty, frequent falling out of fillings. Objecti-vely: all the molars and premolars ofthe lower jaw exhibit I degree mobility.Approximal masticatory surfaces have fi-llings. What splint construction should beapplied in this case?

A. Inlay splintB. Fixed crown splintC. Crown cap splintD. Equator crown splintE. Intradental splint

46. A 35-year-old woman was admi-tetd to the oral surgery department twohours after a road accident. Objectively:cutaneous integuments are pale, there arebeads of sweat on her face. Respirati-on is heavy and obstructed. Pulse rateis 120 bmp, AP is 70/60 mm Hg. Thereis wound penetrating to the oral cavityon the left cheek. During transportationthe woman was in supine position withher head thrown backwards. What kind ofasphyxia is likely to develop in this pati-ent?

A. AspiratingB. ValvularC. StenoticD. ObturativeE. Dislocational

47. Parents of a 4-year-old child complainabout speech defect, namely lallati-on. Examination shows limited tonguemovements, when the tongue is movedforward it turns down, the lower edgeof tongue frenulum is positioned in frontof the submandibular salivary ducts. Thefrenulum is thin and transparent. Specifythe terms of operative intervention:

A. After making the diagnosisB. After complete development of maxi-llofacial bonesC. After eruption of permanent incisorsD. After formation of permanent occlusionE. After eruption of permanent molars

48. A 43-year-old patient complains about

Page 7: Krok 2 - 2010 Question Paper (Stomatology)

Krok 2 Stomatology 2010 7

teeth mobility in the frontal region of hislower jaw, as well as significant cervixesexposure. Objectively: gums in the regi-on of the 44, 43, 42, 41, 31, 32, 33, 34teeth are pale with cyanotic tint. The 42,41, 31, 32 teeth present with I-II degreemobility. The 42, 41, 31, 31 teeth areovercrowded. The 42, 41, 31, 32 teethhave cervix exposure by 1/2 and the 43, 33teeth have cervix exposure by 1/4. Whatorthopaedic construction should be usedin this situation?

A. One-piece guardB. Cast bar Kurliandsky splintC. Cap splintD. Portion crown splintE. -

49. Preventive examination of an 8-year-old boy revealed some lusterless chalk-like spots on the vestibular surface of the11 and 21 teeth, which are localised in theprecervical region. Subjective complaintsare absent. What is the most likely di-agnosis?

A. Acute initial cariesB. White-spotted fluorosisC. Local enamel hypoplasiaD. Acute superficial cariesE. Chronic initial caries

50. A 66-year-old male patient suffersfrom coronary heart disease (CHD) andatherosclerosis. During stomatologicaltreatment the patient complained aboutacute retrosternal pain with irradiationto the left scapula, that was accompani-ed by stupor of the left hand. Objecti-vely: the skin was pale, the sweat stoodout on his forehead. The arterial pressurewas 140/90 mm Hg, pulse was rhythmic 75bpm. The pain wasn’t relieved by Validolbut it started to abate after nitroglycerinintake. What disease provoked the attackin this patient?

A. StenocardiaB. Myocardial infarctionC. Hypertensive crisisD. TachycardiaE. Paroxysmal tachycardia

51. A patient complains about intensivethrobbing toothache in his lower jaw onthe left. He can’t show the aching tooth.It hurts all the time, the pain goes downsometimes, but it becomes stronger asaffected by stimuli. Within the last hoursthe pain provoked by cold stimuli hassomewhat reduced. Objectively: the 17

tooth has a carious cavity, probing causesno pain, the pulp chamber is closed. Whatmethod of treatment is indicated?

A. Vital extirpationB. Devital extirpationC. Devital amputationD. Biological methodE. Vital amputation

52. 3 months after the immediateprosthetics a patient complained aboutthe prosthesis balancing. Objectively:there is a gap between the prosthesis edgeand vestibular surface of alveolar process.What is the tactics of choice?

A. Fabrication of a new prosthesisB. Correction of prosthesis edgesC. Prosthesis relocation with quick-settingplasticD. Dentition buildup with quick-settingplasticE. Correction of dentitions

53. A patient undergoes orthopaedictreatment of bounded edentulous spaceson the upper jaw. He needs fixed full-cast dentures. During his second visit itis required to check whether the internalsurface of the metal framework of thefuture metal-ceramic denture matches thesurfaces of the prepared teeth. In whatway could this be done?

A. In the oral cavity by means of siliconematerialsB. Visually by means of models in thearticulatorC. In the oral cavity by means of tracingpaperD. In the oral cavity by means of a waxplateE. In the oral cavity by means ofstomatoscopic method

54. A 19-year-old girl complains abouthaving crusts, lip tenderness, especi-ally at lip joining. Objectively: thereare yellow-brown crusts on the lip redborder from Klein zone to it’s middle,after their removal bright red smoothsurface without erosions appears. Mucousmembrane in Klein zone is slightlyhyperemic and edematic. What is the mostlikely diagnosis?

A. Exudative form of cheilitis exfoliativaB. Exudative form of cheilitis actinicaC. Epidermolysis bullosaD. Meteorological cheilitisE. Eczematous cheilitis

Page 8: Krok 2 - 2010 Question Paper (Stomatology)

Krok 2 Stomatology 2010 8

55. A 43-year-old patient complains aboutconstant pain in the upper jaw region onthe right, that irradiate to the temple. Thepain was noted one month ago. Objecti-vely: the face is asymmetric because of theswollen right cheek. Mucous membranehas no changes. The breathing throughthe right nasal meatus is obstructed,there are foul-smelling saniopurulent di-scharges. Plan X-ray film of facial skeletonin the semi-axial projection shows intensi-ve opacity of the right maxillary sinusand violated intactness of its interior andsuperior walls. What is the most likely di-agnosis?

A. Upper jaw cancerB. Chronic odontogenic maxillary sinusitisC. Chronic odontogenic osteomyelitisD. Neuritis of the II branch of trigeminusE. Upper jaw osteoma

56. A 44-year-old patient consulted asurgeon about constant acute pain in theupper jaw region on the left that is getti-ng worse during teeth joining. The painwas noted 3 days ago. Objectively: theface is symmetrical, mouth opening isunlimited. The crown fof the 26 toothis half-destroyed. Probing of the cari-ous cavity is painless. Percussion of the26 tooth provokes acute pain. Mucousmembrane of the alveolar process isedematic, hyperemic on the level of the26 tooth. The 26 tooth was treated before.What is your provisional diagnosis?

A. Exacerbation of chronic periodontitisof the 26 toothB. Acute purulent periodontitis of the 26toothC. Acute pulpitis of the 26 toothD. Acute purulent periostitis of upper jawof the 26 tooth on the leftE. Periodontitis of the 26, 27, 28 teeth

57. Parents of a 6-year-old child consulteda dentist about oral cavity sanitation.Objectively: the 85 tooth has a cariouscavity on the distal surface within themantle dentine. Floor and walls are denseand pigmented. Probing is painless. Coldstimuli and percussion cause no pain.What is your provisional diagnosis?

A. Chronic median cariesB. Acute median cariesC. Chronic deep cariesD. Chronic periodontitisE. Chronic fibrous pulpitis

58. A patient complains about acuteconstant pain that is getting worse when

biting down on food, sensation of arecently erupted tooth on the right upperjaw. Examination of the 15 tooth revealeda deep carious cavity communicating withthe dental cavity. Tooth percussion causesacute pain. Probing is painless. Mucousmembrane in projection of the root apex ishyperemic, painful on palpation. X-ray pi-cture shows no changes. What is the mostlikely diagnosis?

A. Acute purulent periodontitisB. Acute diffuse pulpitisC. Exacerbation of chronic periodontitisD. Acute albuminous periostitisE. Acute odontogenic osteomyelitis

59. A 48-year-old male patient has beenwearing partial removable dentures for3 months. According to the patient,results of physical examination and addi-tional methods of testing, the patientwas diagnosed with allergic stomatitisprovoked by dyes of the acrylic resin ofhis prosthesis. Allergic effects of dyes canbe eliminated in the following way:

A. Ffabrication of a denture out ofcolorless plasticB. Fabrication of bilayer basesC. Moulding of plastic by method ofcastingD. Fabrication of cast metal prostheticbasesE. Fabrication of swaged metal bases

60. A 43-year-old patient complains abouta neoplasm in the right submandibularregion that appeared a month ago afterangina. The patient underwent anti-inflammatory therapy but it led to noreduction of the neoplasm. Objectively:body temperature is up to 37, 2oC. Palpati-on reveals a slightly painful, spherical,freely movable, well-defined neoplasm ofdense and elastic consistency in the ri-ght submandibular region. The duct ofsubmandibular salivary gland dischargestransparent saliva. The sublingual plica isunchanged. What is the most likely di-agnosis?

A. Chronic lymphadenitisB. Chronic sialoadenitisC. SialolithiasisD. Adenoma of salivary glandE. Atheroma

61. A patient ordered soldered stainless-steel bridge prostheses. Their fabricati-on involves calibration of crown sleeves.Which device is used for this purpose?

Page 9: Krok 2 - 2010 Question Paper (Stomatology)

Krok 2 Stomatology 2010 9

A. SamsonB. Parker’sC. Larin’sD. Cope’sE. Bromshtrom

62. A 20-year-old patient complains aboutunaesthetic look of the 24 tooth fromthe moment of its eruption. Objectively:enamel of the 24 tooth is partly absent,the dentine is yellow. The 64 tooth wastreated more than once when she was achild, but because of frequent exacerbati-ons, edemas and gingival fistula the 64tooth was extracted when she was 9 yearsold. What is the most likely diagnosis?

A. Localised hypoplasiaB. Initial cariesC. FluorosisD. Systemic hypoplasiaE. Enamel aplasia

63. A 50-year-old female patient complai-ns about sensation of tightness ofbuccal mucosa and roughness of thelateral surface of tongue. The patientundergoes regular check-up at a di-spensary department for compensatedform of diabetes mellitus. Objectively:there are white and grey areas in form oflacy pattern on the buccal mucosa on theright and on the lateral surface of tongue.The surface of affected region cannot bescraped off. What is the most likely di-agnosis?

A. Lichen ruber planusB. Secondary syphilisC. Lupus erythematosusD. Pseudomembranous candidiasisE. Leukoplakia

64. A 45-year-old patient consulted adentist about extraction of the 13 tooth.What tools should be applied for extracti-on?

A. Straight forcepsB. S-shaped forceps curved rightC. Bayonet-shaped forcepsD. Straight elevatorE. S-shaped forceps

65. According to the mother, a 5-year-old child complains about pain duringswallowing, weakness, body temperaturerise upt to 39, 5oC, swelling of submentallymph nodes. Objectively: the child’scondition is grave, body temperature is38, 8oC. Mucous membrane of oral cavityis brightly hyperemic and edematic withhaemorrhages and ulcerations. Pharynx is

brightly hyperemic, lacunae are enlargedand have necrosis areas. Regional, cervi-cal, occipital lymph nodes are painful,enlarged and dense. What is the most li-kely diagnosis?

A. Infectious mononucleosisB. Acute herpetic stomatitisC. Necrotizing ulcerative gingivostomatitisD. Herpetic anginaE. Lacunar tonsillitis

66. A 28-year-old patient suffers from adisease without prodromal manifestationsthat declares itself through oral mucosalesion consisting of 1-2 roundish elements5-8 mm large which are circumscribed bya hyperemic rim and covered with yellow-grey coating. The disaese recurrence isobserved quite regularly 3-4 times a year.These presentations are typical for thefollowing disease:

A. Chronic recurrent aphthous stomatitisB. Lichen ruber planusC. Chronic herpes recidivicusD. Erythema multiformeE. Papular syphilis

67. A 48-year-old female patient complai-ns about some discomfort induced bywearing an upper jaw clasp denture wi-th elastic attachments. The patient notesthe denture mobility during masticationand articulation. The patient asks to fabri-cate a new denture with considerationof the indicated defects. Clinical crownsof abutment teeth are low. What fixationshould be the most efficient in this case?

A. TelescopicB. Arch-barC. LockingD. Saddle-typeE. Clasp

68. A 75-year-old patient ordered acomplete removable denture for theupper jaw. While adjusting the individualtray by Herbst’s method the orthodontistrevealed that the tray could be thrown offwhen the patient rounded his lips. Whatpart of the tray requires correction?

A. Vestibular part between the caninesB. Distal part along the A lineC. In the region of buccal cordsD. Along the whole vestibular edgeE. Behind the maxillary tuber

69. A 36-year-old woman complains aboutexperiencing lip dryness and desquamati-on for a month. Application of indifferent

Page 10: Krok 2 - 2010 Question Paper (Stomatology)

Krok 2 Stomatology 2010 10

ointments is ineffective. Objectively: redborder of lower lip is of rich red color,it is moderately infiltrated, covered withclosely adhering greyish scales, it bleedsand hurts in the attempt to remove them.Opacification of epithelium in form ofwhite stripes is present in the nidus peri-phery, there is also an area of depressionin the centre. What is the most likely di-agnosis?

A. Lupus erythematosusB. Commissural cheilitisC. Lichen ruber planusD. LeukoplakiaE. Cheilitis exfoliativa

70. A 30-year-old patient complains aboutpain in the upper jaw tooth caused bysweet and cold food, that was first regi-stered one month ago. Objectively: the 26tooth has a carious cavity within mantledentine. Dentine of its walls is softened.Probing along the enamel-dentine juncti-on is painful. Percussion is painless.Thermodiagnosis causes pain, that quicklyabtes after elimination of stimulus. Whatis the most likely diagnosis?

A. Acute median cariesB. Acute initial cariesC. Pulp hyperaemiaD. Acute deep cariesE. Chronic fibrous pulpitis

71. A 55-year-old patient has a repeatedappointment with a dentist. He hasordered complete removable lamellardentures for both upper and lower jaws.During his previous visit the dentisttook complete anatomic impressions ofboth jaws and sent them to the dentalmechanic. What manipulations should thedentist perform at the next clinical stage?

A. Adjust individual traysB. Locate and fix central occlusionC. Check the prosthesis constructionwithin the oral cavityD. Correct the prosthesis and give thepatient necessary instructionsE. Fabricate occlusal rims

72. A 47-year-old patient complains abouta sensation of foreign body on his tongue,discomfort during talking, oral cavitydryness. Objectively: there are dark fili-form papillae up to 5 mm long on theback of tongue. What is the most likelydiagnosis?

A. GlossophytiaB. Median rhomboid glossitisC. Benign migratory glossitisD. Fissured tongueE. Acute glossitis

73. A patient complains about destroyedcrown of the 27 tooth. The patient was di-agnosed with chronic periodontitis of theindicated tooth. Objectively: the crown ofthe 27 tooth is destroyed completely. It isnecessary to extract this tooth. What fi-eld block anesthesia should be applied foroperation?

A. Tuberal and palatinalB. InfraorbitalC. TuberalD. Infraorbital and tuberalE. Tuberal and incisor

74. A 32-year-old patient complains abouta cosmetic defect. Objectively: the crownsof 12, 11, 21, 22 teeth are destroyed bycaries by 2/3 of their height. The occlusi-on is orthognathic. X-ray picture showsthat root canals are filled to the top. Thereare no patological changes in periapicaltissues. What construction should be themost efficient in this situation?

A. Stump inlays and metal-ceramic crownsB. Artificial metal swaged crownsC. Restoration of tooth crowns withcomposite materialsD. Fabrication of plastic inlaysE. Richmond’s pivot crowns

75. A 25-year-old male patient came to thetraumatology. He complains about pain inthe right half of his face, limited mouthopening. Objectively: there is a moderateedema and haematoma in the region ofthe right malar arch. Palpation shows aslightly painful impaction of bone tissuein the region of the right malar arch. Thebite isn’t disturbed. Amplitude of mouthopening is 1,5 cm. At making an attemptto open the mouth wider the patient feelsa mechanical obstacle and pain intensifi-cation. What is the most likely diagnosis?

A. Fracture of the right malar archB. Fracture of the right zygomatic boneC. Traumatic arthritis of TMJD. Le Fort III maxillary fracture (upper)E. Le Fort II maxillary fracture

76. The patient is 75 years old. Theinspection of construction of completeremovable dentures revealed that onlylateral artificial teeth were in contact,there was a gap between frontal teeth.

Page 11: Krok 2 - 2010 Question Paper (Stomatology)

Krok 2 Stomatology 2010 11

In the lateral part on one side cusp-to-cusp contact was present, on the other sidethere was a horizontal gap; the central li-ne was deviated. What mistake was made?

A. The patient was thought to have lateralocclusionB. Occlusal rims were irregularly softenedC. The patient was thought to have anteri-or occlusionD. Interalveolar height was increasedE. Wax moulds were deformed whiledetermining central occlusion

77. A 5-year-old child sustained a dentalinjury. Objectively: the crowns of the 51,61 teeth are shorter than neighbouri-ng teeth by 1/2. Mucous membrane isedematic and hyperemic in theregion ofthe 51, 61 teeth. X-ray picture shows thatthere is no periodontal fissure in the apicalparts of roots of the 51, 61 teeth, apexesof the 51, 61 teeth are imbedded intothe spongy substance of body of maxilla.What treatment tactics would be the mostefficient?

A. Extraction of the 51, 61 teethB. Regular medical check-upC. Reposition of the 51, 61 teethD. Ligature splinting of the 51, 61 teethE. Reimplantation

78. A 5-year-old child had to undergoan operated for ankyloglossya. Therewere no external symptoms of preoperati-ve anxiety. Aafter injection of 1,5 mlof 0,5% novocaine solution the childpresented with motor anxiety, vomituri-tion, stomachache. Objectively: the chi-ld is conscious, face and neck skin ishyperemic, tachypnoe is present, pulse israpid. What is the most likely diagnosis?

A. Allergic reaction to novocaineB. Pain shockC. Cardiovascular collapseD. GiddinessE. Overdosage of anesthetics

79. A 19-year-old patient complains aboutpain in the submental part of mandible.The day before he got a trauma. Objecti-vely: there is a slight swelling of tissuesin the mental area. The mouth can beopened widely enough. All the teeth areintact. Mucous membrane is edematic inthe region of central incisors, it bleeds sli-ghtly. In this region mobility of mandiblefragments is present. Occlusion is undi-sturbed. What splint should be chosen bythe dentist?

A. Smooth braceB. Splint with spacer barC. Splint with guide planeD. Anchor splint with intermandibularfixationE. Ivy loops for 31, 32, 42, 41

80. A 27-year-old woman has been treatedfor pulpitis by method of devital extirpati-on. Arsenic paste was left in the 15 tooth.The patient came to see a dentist for thesecond time only on the fourth day afterher previous visit. She was diagnosed withtoxic periodontitis. What is the most opti-mal agent for treatment of root canal inthis case?

A. UnithiolB. CresopheneC. TripsinD. EugenolE. Hydrocortisone emulsion

81. A 37-year-old female patient complai-ns about pain, gingival haemorrhage,halitosis, body temperature rise upto 37, 2oC. Objectively: gums areapparently hyperaemic, edematic, bleedeasily, parodontal pouches are 3-4 mm deep and contain purulentexudate. Orthopantomogram shows di-ffuse osteoporosis of alveolar process,resorption of interdental septa down to1/3 of their height. What is the most likelydiagnosis?

A. Exacerbation of generalized I degreeperiodontitisB. Exerbation of chronic catarrhal gingivi-tisC. Chronic generalized I degree peri-odontitisD. Chronic generalized II degree peri-odontitisE. Exacerbation of generalized II degreeperiodontitis

82. A 24-year-old woman complainsabout severe pain in the mouth, bodytemperature up to 38oC, indisposition.The same condition occurs periodicallyfor several years after catching a cold.Objectively: the lips are covered withbloody crusts, there are opened bladdersand erosions, covered with fibrinogenousdeposit on mucous membrane of lipsand cheeks that is apparently hyperemicand edematic. Hypersalivation is present.What is the most likely diagnosis?

Page 12: Krok 2 - 2010 Question Paper (Stomatology)

Krok 2 Stomatology 2010 12

A. Erythema multiformeB. Pemphigus vulgarisC. Dermatitits multiformis, Duhring’sdiseaseD. Nonacantholytic pemphigusE. Chronic herpes recidivicus

83. A 30-year-old patient complainsabout a carious cavity. Objectively: the16 tooth is discoloured and has a deepcarious cavity communicating with thedental cavity. Probing, percussion causeno pain. Electroodontodiagnosis is 100microampere. X-ray picture shows wideni-ng of periodontal fissure. What is the mostlikely diagnosis?

A. Chronic fibrous periodontitisB. Chronic granulating periodontitisC. Chronic granulomatous periodontitisD. Chronic gangrenous pulpitisE. Chronic deep caries

84. A 65-year-old patient complainsabout unsatisfactory fixation of completeremovable lamellar denture of his upperjaw. The denture was fabricated 6 yearsago. Objectively: balancing and poor fi-xation of complete removable denture ispresent. What is the reason of such condi-tion?

A. Atrophy of osseous base of theprosthetic bed tissuesB. Wear of artificial teethC. Bad hygiene of removable dentureD. Discoloration of basic plasticE. Loss of certain antagonists

85. A 10-year-old boy complains aboutmissing teeth. Objectively: the face issymmetrical, disproportional because ofshortening of the lower third. In the oralcavity: the 12, 14, 15, 17, 22, 24, 25, 27, 34,35, 37, 44, 45, 47 teeth are missing. X-raypicture shows partial adentia and absenceof some tooth germs. Choose the mostefficient prosthetic device:

A. Partial removable lamelalr prosthesisfor both jawsB. Bridge prosthesesC. Clasp dental prosthesesD. Cantilever dental bridgesE. The defect should be restored byimplants

86. A child was born with body weight at arate of 3200 g and body length at a rate of53 cm, 9 points on Apgar score. It was thefirst physiological delivery. What positi-on of child’s mandible is usually observedafter birth?

A. Physiological retrogeniaB. Physiological progeniaC. Central occlusionD. Direct relationE. Posterior occlusion

87. A 3-month-old child has been indisease state for two days. The chi-ld is anxious, refuses food, has normalbody temperature. Objectively: mucousmembrane of oral cavity is edematic andhyperemic. There is white caseous coati-ng on the back of tongue and buccalmucosa. After the coating removal onecan see extremely hyperemic surface withpetechial haemorrhages. What is the mostlikely diagnosis?

A. Acute candidal stomatitisB. Mild leukoplakiaC. Acute herpetic stomatitisD. Herpetic anginaE. Lichen ruber planus

88. A 35-year-old patient consulted adentist about extraction of the 14 toothbecause of exacerbation of chronic peri-odontitis following ineffective therapeutictreatment. What tools should be appliedfor extraction?

A. S-shaped forcepsB. Crown bayonet-shaped forcepsC. S-shaped forceps curved rightD. Straight forcepsE. Root bayonet-shaped forceps

89. The 12, 22 teeth of an 8-year-old childare missing. There is not enough space indentition for them. X-ray picture showsno tooth germs. The 12 tooth of the chi-ld’s father is missing and the 22 toothis conoid. What is the reason for suchpathological changes?

A. Hereditary adentiaB. CariesC. Extraction of teethD. TraumaE. Rachitis

90. A 35-year-old patient complains aboutconstant dull pain in the 25 tooth that isgetting worse when biting down on food.Objectively: masticatory surface of the 25tooth has a carious cavity communicati-ng with the dental cavity. The purulentdischarges from the canal followed theprobing. What method of diagnosticsshould be applied to confirm the di-agnosis?

Page 13: Krok 2 - 2010 Question Paper (Stomatology)

Krok 2 Stomatology 2010 13

A. X-ray examinationB. Electric pulp testC. Thermal testD. Bacteriological examinationE. Deep probing

91. After complex extraction of the 37tooth a patient experienced anaesthesiaof the left half of his lower lip and chin.Electroodontodiagnos showed reductionof lower jaw teeth electroexcitability onthe left. What is the most likely diagnosis?

A. Neuritis of the left inferior alveolarnerveB. Neuralgia of the left inferior alveolarnerveC. Alveolitis in the region of socket of the37 toothD. Herpes Zoster n.TrigeminiE. Acute osteomyelitis of mandible body

92. A female patient consulted a denti-st about dental prosthetics. Objectively:the lower third of the patient’s face isshortened, nasolabial folds are deepened,frontal group of lower and upper jaw teethis missing, the crowns of the 17, 15, 26, 27,36, 37, 45, 47 teeth are worn off by 2/3of their height, masticatory surfaces ofthese teeth are smooth and pigmented.The alveolar process isn’t hypertrophied.Interalveolar height is decreased. Specifythe form of pathological tooth wear in thispatient:

A. Horizontal, noncompensated, III gradeseverityB. Horizontal, compensated, II gradeseverityC. Vertical, noncompensated, III gradeseverityD. Vertical, compensated, III grade severi-tyE. Combined, noncompensated, III gradeseverity

93. A 25-year-old man consulted a dentistabout extraction of the 18 tooth, it’s crownis destroyed by 1/2. The tooth was treatedmore than once. During tooth extractionthe maxillary tuber was accidentally tornoff. What actions should be taken?

A. To remove the fragment and stitch upthe woundB. To try to restore the fragment to itsplaceC. To restore the fragment to its place andfix it thereD. To remove the fragmentE. To remove the fragment and tamponthe wound

94. Preventive examination of a 4-year-old child revealed a deep carious cavi-ty on the masticatory surface of the 54tooth. The cavity has no intercommuni-cation with the tooth cavity and is filledwith dense dentine. Probing, percussion,thermal test of the 54 tooth provoke nopain. The decay/filled index is 1, hygieneindex is 1,9. What filling material is themost suitable for permanent filling of the54 tooth?

A. Glass-ionomer cementB. Silicate cementC. Silicophosphate cementD. Chemical compositeE. Photopolymer composite

95. A 34-year-old male patient complai-ns about acute spasmodic pain in theregion of his upper jaw on the left thatis getting worse as affected by cold sti-muli. Toothache irradiates to the ear andtemple. He had acute toothache of the 37tooth one year ago, but he didn’t consulta dentist. Pain recurred three days ago.Objectively: the 37 tooth has a cariouscavity communicating with the dentalcavity. Probing of the opened cariouscavity is extremely painful. X-ray pictureshows widening of periodontal fissure atthe root apex of the 37 tooth. What is themost likely diagnosis?

A. Exacerbation of chronic pulpitisB. Exacerbation of chronic granulatingperiodontitisC. Exacerbation of chronic fibrous peri-odontitisD. Acute diffuse pulpitisE. Acute purulent pulpitis

96. A 69-year-old patient underwent anoperation for complete removal of hislower jaw. They took impressions ofboth jaws and fabricated a lower jawreplacement prosthesis before operation.What elements will provide fixation of thisprosthesis in the oral cavity?

Page 14: Krok 2 - 2010 Question Paper (Stomatology)

Krok 2 Stomatology 2010 14

A. Foshar’s spiral springsB. Wire loopsC. ClaspsD. MagnetsE. Attachments

97. Parents of a 7-year-old boy complainabout missing of the 11 tooth. Objectively:there is enlargement of alveolar processin projection of the 11, 12 teeth. X-ray pi-cture shows multiple shades of differentsize. They have dentate shape and looklike hard tooth tissues. What is the mostlikely diagnosis?

A. Odontoma of maxillaB. Follicular cyst of maxilla starting fromthe 11 toothC. Cementoma of maxillaD. Adamantinoma of maxillaE. Odontogenic fibroma of maxilla

98. A 47-year-old patient complains aboutdiscolouration of the 11, 12 teeth. Objecti-vely: the 12, 12 are changed in colour,canals are filled to the top. It was deci-ded to make metal-ceramic crowns for the11, 12 teeth. What is the optimal angle forthe preparation of approximal surfaces ofabutment teeth?

A. 5-8 degreesB. 15-20 degreesC. 10-15 degreesD. 20-25 degreesE. 30-35 degrees

99. After preventive examination a10-year-old child was diagnosed withosteoporosis circumscripta of the 13, 12,11, 21, 22, 23 teeth. The patient was admi-nistered electrophoresis with reminerali-zing solutions. What preparations and inwhat order are to be applied in this case?

A. Calcium and posphorus preparationswith the following application of fluorinepraparationB. Fluorine preparations with the followi-ng application of calcium and phosphoruspreparationsC. Calcium preparations onlyD. Phosphorus preparations onlyE. Fluorine preparations only

100. A 12-year-old male patient consultedan orthodontist about odontoloxia.Objectively: the face is symmetricand proportional. In the oral cavity:permanent occlusion, occlusal relati-onship is orthognathic in the lateral parts,the 13 tooth is located off dentition on thepalate, biometric measurements show that

the width of the 13 tooth is 11,4 mm, thedistance between the 12 and 14 teeth is 4,6mm, the width of the 14 tooth is 7,6 mm.Suggest the treatment plan:

A. Extract the 14 tooth and move the 13tooth into it’s placeB. Open the bite and move the 13 toothinto its due placeC. Move the 13 tooth into its due placewithout bite openingD. No treatment is requiredE. Extraction of the 14 tooth

101. Parents of an 8-year-old childcomplain about a painful formation inthe child’s oral cavity that obstructs foodintake. The same complaints were regi-stered two years ago. Mucous membraneof lateral tongue surface is hyperemic andedematic. There is an oval erosion over0,7 cm large covered with yellow greyi-sh deposit. Erosion edges are hyperemicand painful on palpation. The child has ahistory of chronic cholecystocholangitis.What is the most likely diagnosis?

A. Chronic recurrent aphthous stomatitisB. Erythema multiformeC. Behcet’s syndromeD. Stevens-Johnson syndromeE. Traumatic erosion

102. A 50-year-old patient complainsabout a cosmetic defect of the upperfrontal teeth. After meticulous examinati-on the patient was diagnosed with wedge-shaped defect of the 11, 12, 13, 21, 22, 23teeth. What material should be chosen forthe tooth filling?

A. Compomer restorative materialB. Silicate cementC. Glass-ionomer cementD. Chemical-cure compositeE. Silicophosphate cement

103. A 12-year-old child complains aboutbleeding and pain in the 46 tooth duri-ng eating. He has a history of acute painsome time before. Objectively: there isa deep carious cavity (Black’s class II)communicating with the dental cavity,partially filled with overgrown pulp. Pulptissue is bleeding, painful on touch. Thereis soft white dental deposit. What is themost likely diagnosis?

Page 15: Krok 2 - 2010 Question Paper (Stomatology)

Krok 2 Stomatology 2010 15

A. Chronic hypertrophic pulpitisB. Chronic papillitisC. Chronic simple pulpitisD. Chronic gangrenous pulpitisE. Chronic granulating periodontitis

104. Examination of a 6-year-old girlrevealed a deep carious cavity in the 85tooth. Percussion and probing are pai-nless. After removal of the softened denti-ne, communication with the tooth cavityshowed up. Deep probing is painless. X-ray picture of the 85 tooth shows the focusof destruction of bone tissue in the regionof bifurcation; cortical plate of the 35 hasno pathological changes. It is most expedi-ent to use the following material for theroot filling:

A. Zinc oxide eugenol cementB. Resorcin-formalin pasteC. Glass-ionomer cementD. Phosphate cementE. Calcium-containing paste

105. A 33-year-old male patient complai-ns about tongue pain that is getting worseduring eating and talking. Objectively:there is a painful ulcer 0,6 cm large onthe lateral surface of tongue. The floor iscovered with grey deposit. The crown ofthe 47 tooth is destroyed. What is the mostlikely diagnosis?

A. Decubital ulcerB. Trophic ulcerC. Hard chancreD. Tuberculous ulcerE. Cancerous ulcer

106. A 27-year-old patient complainsabout pain of the 35 tooth caused by coldstimuli, that quickly abates after elimi-nation of stimulus. Objective examinationof the 35 tooth revealed a defect of hardtooth tissue within enamel. Defect edgesare fragile and white. Electroodontodi-agnosis is 5 microampere. What is themost likely diagnosis?

A. Acute superficial cariesB. Local enamel hypoplasiaC. Acute initial cariesD. Enamel necrosisE. Enamel erosion

107. A 13-year-old patient complainsabout gingival haemorrhage during toothbrushing. Objectively: gums around all theteeth are hyperemic and edematic, PMAindex (papillary marginal alveolary index)is 46%, Greene-Vermillion hygiene indexis 2,5. Provisional diagnosis: exacerbation

of chronic generalized catarrhal gingivitis.This patient should be recommended touse a toothpaste with the following activecomponent:

A. ChlorhexidineB. Calcium glycerophosphateC. MonofluorophosphateD. Vitamins A, D, EE. Microelement complex

108. Examination of a male patientrevealed that a tumour occupied themajor part of his tongue; tongue mobilitywas limited, there were solitary regionalnodes in the region of affection. Afterthe cytological analysis the patient was di-agnosed with cancer of tongue T2N1M0.What treatment should be chosen for thispatient?

A. Combined methodB. Radiation therapyC. Surgical removal of tumorD. ChemotherapyE. Removal of lymph nodes

109. A 22-year-old patient complainsabout short-lasting spasmodic pain inthe 26 tooth with long periods of painabsence. Objectively: the 26 tooth has acarious cavity in the masticatory surface,dental cavity is not opened. Probingcauses pain in the pulp horn projecti-on. Thermal stimuli cause sharp pain,percussion is painless. Regional lymphnodes are not palpable. What is the mostlikely diagnosis?

A. Acute circumscribed pulpitisB. Acute deep cariesC. Acute diffuse pulpitisD. Acute purulent pulpitisE. Exacerbation of chronic pulpitis

110. A 27-year-old patient complainsabout gingival haemorrhage during toothbrushing. Objectively: gingival edge ishyperaemic, edematic, painful on touch.Periodontal pouches are absent. X-ray pi-cture shows resorption of cortical plate,osteoporosis of interdental septa. What isthe most likely diagnosis?

A. Initial generalized periodontitisB. Exacerbation of chronic catarrhal gingi-vitisC. Generalized I degree periodontitisD. Chronic catarrhal gingivitisE. Periodontitis

111. A 25-year-old male patient gotbilateral fracture of the lower jaw. A

Page 16: Krok 2 - 2010 Question Paper (Stomatology)

Krok 2 Stomatology 2010 16

fragment in the region of the 44, 43, 42, 41,31, 32, 33, 34 teeth is displaced downwardand backward. What appliance should beused for the fragment reposition?

A. Post’s applianceB. One-jaw bite-guard splintC. Kurlyandsky’s appliance with leversD. Weber’s applianceE. Shur’s appliance

112. A 9-year-old child complains aboutpain caused by sweet and sour food in anupper tooth on the left. Objectively: the26 tooth has a carious cavity on the masti-catory surface within the enamel limits.What is the optimal material to fill the 26tooth?

A. СompositeB. Glass ionomerC. Silicophosphate cementD. Silicate cementE. Zinc phosphate cement

113. A 57-year-old patient complainsabout dental hypersensitivity to chemicalstimuli, gum itch. Objectively: roots areexposed down to 1/3 of their length, gumsare dense and pale rose. Wedge-shapeddefects in the area of the 14, 13, 24, 25,26, 34 teeth are within the dentine. Probi-ng of dental cervixes and wedge-shapeddefects is painfull. What is the most likelydiagnosis?

A. I degree parodontosisB. Generalized periodontitisC. II degree periodontitisD. Atrophic gingivitisE. Localised periodontitis

114. A 7-year-old child was diagnosedwith chronic periodontitis of the 64, 85teeth. The child suffers from hemophiliaA. Specify the treatment tactics:

A. Extraction of teeth in hematologicaldepartment after due pretreatmentB. Extraction of teeth is possible in bothin-patient and out-patient hospitalsC. Extraction of teeth can be performedin an out-patient hospital with thefollowing socket tamponade by meansof haemostatic spongeD. Extraction of teeth can be performed inthe oral surgery departmentE. Extraction of teeth is contra-indicatedbecause of high risk of haemorrhage

115. Preventive examination of a 6-year-old child revealed temporary teeth bi-te. Upper and lower dental arches are

trapeziformed. Upper incisors overlaplower incisors more than by 2/3. Incisorsand second molars are in the same relati-on. There is no space between frontalteeth. Upper dental arch is bigger thanlower dental arch by the cheek tuberclesize. Bite abnormality is observed in thefollowing planes:

A. Sagittal and verticalB. Sagittal and lateralC. Sagittal and occlusalD. Sagittal and nasalE. Sagittal and frankfurt

116. Preventive examination of a 9-year-old girl revealed broad bridgeof nose, narrow nasal passages, half-opened mouth, problems with lip joini-ng, elongated lower third of face. Thechild presents with transitional occlusion.There is vertical gap 4-5 mm large fromthe 53 to the 64 tooth in the frontal regi-on. Relationship of the first permanentmolars complies with Angle’s class I. Thechild pronounces hissing sounds indisti-nctly. Specify the most likely factor ofocclusion deformation:

A. Nasal respiration disorderB. Tongue parafunctionC. Tongue suckingD. Infantile swallowingE. There is no correct answer

117. A 40-year-old patient is hospitali-zed with Le Fort II maxillary fracture.The teeth are intact. Choose the mostappropriate apparatus for this patient:

A. Standard Zbarzh constructionB. Yadrova’s apparatusC. Rudko’s apparatusD. Yermolaev-Kulagov’s apparatusE. Gunning-Port’s splint

118. A 57-year-old patient complainsabout a slowly growing swelling in thesublingual region. She noted it 3 monthsago. Objectively: there is an elastic swelli-ng in the sublingual region. The swellingis painless, mucous membrane over it iswith tints of blue. What is the most likelydiagnosis?

A. RanulaB. SalivolithiasisC. Dermoid cyst of oral cavity floorD. Lipoma of sublingual areaE. Hemangioma of sublingual area

119. A 38-year-old patient consulteddental surgery about extraction of the 36

Page 17: Krok 2 - 2010 Question Paper (Stomatology)

Krok 2 Stomatology 2010 17

tooth. What kind of anesthesia should beapplied for extraction?

A. TorusB. MandibularC. TuberalD. InfiltrationE. Voino-Yasenetsky truncal

120. A patient ordered partial removablelamellar dentures for the upper and lowerjaw. An orthodontist took elastic alginateimpressions of both jaws. What is his nextstep?

A. To send the impressions for disinfectionB. To let the impressions dry out in theopen airC. To invite a dental mechanic for jointanalysis of the impressionsD. To send the impressions immediately tothe laboratoryE. To put the impressions into the microtenbag for 90 minutes

121. A 42-year-old woman came toprosthetic dentistry to make dentalprosthetics. Objectively: dental formula is

18 . . . . 13 12 11 21 22 23 . . . . 2848 47 46 45 44 43 42 41 31 32 33 34 35 36 37 .

The bite is deep, clinical crowns are low,survey line is not marked. The patientsuffers from epileptiform attacks. Whatprosthesis is indicated?

A. Partial removable lamellar prosthesiswith metal baseB. Dental bridgesC. Partial removable lamellar plasticprosthesis with retaining claspsD. Partial removable lamelalr prosthesiswith supporting-retaining claspsE. Clasp denture

122. A 58-year-old patient was diagnosedwith fracture of lower jaw with formati-on of a false joint. Objectively: the 38, 36,32, 41, 43, 48 teeth are missing. Preservedteeth are intact, stable. There is no di-splacement of lower jaw fragments. X-raypicture shows a bone tissue defect to up 2cm large. What construction of prosthesisis indicated in this situation?

A. Fixed Oxman’s prosthesis with pivotpointB. Oxman’s single-jointed prosthesis withpivot pointC. Gavrilow’s prosthesis with pivot pointD. Kurlyandsky prosthesis with rollerdamper claspE. Oxman’s double-jointed prosthesis withpivot point

123. A 39-year-old patient complainsabout experiencing pain in the regionof the 21 tooth for 2 days. It is knownfrom the anamnesis that the indicatedtooth was treated for carious. Objecti-vely: the 21 tooth is covered with metal-ceramic crown, mucous membrane inapex projection is hyperaemic. Percussionof the tooth is sharply painful. X-ray pi-cture shows improperly filled root canal. Itis planned to take off the 21 tooth crown.What kind of anesthesia should be appli-ed?

A. Field block anesthesiaB. Infiltration anesthesiaC. Intraligamentous anesthesiaD. Application anesthesiaE. Intraosseous anesthesia

124. A 8-year-old child has a deep cariouscavity communicating with dental cavi-ty on the distal-approximal masticatorysurface of the 75 tooth. Probing causespain. Percussion is painless. Cold watercauses slowly abating pain. The toothdecayed some months ago and wasn’ttreated. What treatment method is effi-cient in this case?

A. Devital amputationB. Biological methodC. Vital extirpationD. Vital amputationE. Devital extirpation

125. A patient consults a prosthodonti-st about a dentition defect of the 14, 15,16 teetht. It is necessary to prepare tethfor crowning. Before the preparation thehandpiece must be processed with:

A. With 3% solution of chloramine (twotimes with 15 minute interval)B. With 3% solution of hydrogen oxideC. With 3% solution of chloramine onceD. With solution of potassiumpermanganateE. With furacilin solution

126. A 36-year-old female patientcomplains about acute pain, clicking inthe right temporomandibular joint, burni-

Page 18: Krok 2 - 2010 Question Paper (Stomatology)

Krok 2 Stomatology 2010 18

ng pain in the region of the right externalacoustic meatus. Movements of lower jaware step-like with short-standing disabli-ng moments and acute pain in the joi-nt. Objectively: the face is symmetric.Occlusion is orthognathic. Palpation oflateral pterygoid muscles is painful onthe right. Tomogram shows that bonestructure circuits of joints are smooth andslick. What is the most likely diagnosis?

A. Dysfunction of TMJB. Rheumatic arthritis of TMJC. Acute posttraumatic arthritis of TMJD. Deforming arthrosis of TMJE. Ankylosis of TMJ

127. A 45-year-old teacher consulted aprosthodontist about choosing efficientprosthetic tactics. Objectively: the 12, 11,21, 22 teeth present with III degree mobi-lity. What orthopaedic care is the mostefficient?

A. Immediate prostheticsB. Early prostheticsC. Fixed prosthesisD. Delayed prostheticsE. Clasp denture

128. A 62-year-old female patientconsulted a dentist about orthopaedictreatment of the 12, 21, 22 teeth. Thearterial pressure was 165/110 mm Hgbefore prosthetic procedure. Because ofemotional stress arterial pressure rose upto 220/130 mm Hg. What is the most likelydiagnosis?

A. Hypertensive crisisB. Acute respiratory failureC. Acute heart failureD. SyncopeE. Collapse

129. A 16-year-old patient complainsabout experiencing gingival enlargement,pain and haemorrhage throughout a year.Objectively: gingival papillae are enlargedand overlap tooth crowns by 1/2 of theirheight. Gums exhibit bright red granulati-ons and are painful on palpation. Thereare considerable calculus deposits. X-ray picture shows no changes of alveolarprocess. What is the most likely diagnosis?

A. Hypertrophic gingivitisB. Localised periodontitisC. Ulcerous gingivitisD. Generalized periodontitisE. Catarrhal gingivitis

130. Examination of a 25-year-old male

patient revealed maximal number ofocclusional contacts of opposing teethduring denture joining. X-ray pictureshows that the articular head of the lowerjaw is near the base of the slope of arti-cular tubercle. What type of occlusion ispresent?

A. Central occlusionB. Anterior occlusionC. Right lateral occlusionD. Left lateral occlusionE. Posterior occlusion

131. A 49-year-old male patient complainsabout gingival haemorrhage, teeth mobi-lity in the frontal region of his lower jaw,hypersensitivity of dental cervixes. X-raypicture shows widening of periodontal fi-ssure in the region of the 42, 41, 31, 32teeth, as well as resorption of alveolarprocess tissue to 1/3 of root length. The42, 32 teeth present with I degree mobili-ty, the 41, 31 teeth present with II degreeof teeth mobility. What temporary splintshould be used for frontal stabilization ofdentition?

A. NovotnyB. MamlockC. TreumanD. KoganE. Kopeinin

132. For fabrication of an externalprosthesis a 62-year-old male pati-ent needs "Hippocratic facies". Whatimpression material should be applied?

A. GypsumB. DentafolC. StensD. StomaflexE. Repin

133. A 67-year-old patient was diagnosedwith cemental caries of the 35 tooth. Whatfilling material should be chosen?

A. Glass-ionomer cementB. Silver amalgamC. Chemical-cure composite materialD. SilidontE. Silicin

134. A 47-year-old patient complainsabout limited mobility of her lower jawin the morning; periodical dull pain inthe right temporomandibular joint (TMJ)and general joint stiffness. Accordingto the patient, the stiffness disappearsthroughout the day after joint "exerci-sing". Objectively: the patient’s face is

Page 19: Krok 2 - 2010 Question Paper (Stomatology)

Krok 2 Stomatology 2010 19

symmetric, mouth opening is limiteddown to 2,5 cm, there is also joint clicki-ng. Median line deviates to the right by3-4 mm, palpation of the right articularhead is painful. What is the most likely di-agnosis?

A. Arthrosis of the right TMJB. Acute serous arthritis of the right TMJC. Chronic arthritis of the right TMJD. Fracture of the right condyle of mandi-bleE. Right-sided anterior dislocation ofmandible

135. A 10,5-year-old child complainsabout painful skin rash on his lips.Objectively: red border is edematic andhyperemic, covered with cracks and multi-ple bloody crusts. There are small vesi-cles with serous contents on the upper lipskin that merge together in certain areas.Maceration and madescence of skin is alsopresent, especially in the mouth corners.What is the most likely diagnosis?

A. Eczematous cheilitisB. Meteorological cheilitisC. Atopic cheilitisD. Actinic cheilitisE. Exfoliative cheilitis

136. An injured man was hospitalized tothe oral surgery department with ruptureof soft tissues of palate and pharynx. Inprocess of respiration the dangling flapfrom the wound partially or completelyblocks the larynx aperture. What kind ofasphyxia may develop in this patient?

A. ValvularB. AspiratingC. ObturativeD. DislocationalE. Stenotic

137. A 27-year-old female patient ispregnant, duration of gestation is 21weeks. She has indication for extractionof the 28 tooth because of exacerbationof chronic periodontitis after ineffecti-ve therapeutic treatment. What kind ofanaesthesia should be used for toothextraction?

A. Tuberal and palatinalB. Tuberal and incisorC. Infraorbital and palatinalD. TorusE. Mandibular

138. A 50-year-old patient complainsabout difficult mastication, teeth mobili-

ty, offensive breath, gingival hemorrhage.Objectively: the gum is hyperemic andcyanotic, dental calculus is present.Parodontal pouches are 8 mm deep wi-thin upper jaw molars. The pouches ofother teeth are 6 mm deep. X-ray pictureshows resorption of bone tissue by 2/3-1/2of tooth roots. What is the most likely di-agnosis?

A. Chronic generalized III degree peri-odontitisB. Chronic generalized II degree peri-odontitisC. Chronic generalized I degree peri-odontitisD. Acute generalized III degree peri-odontitisE. Acute generalized II degree periodonti-tis

139. A 20-year-old patient complai-ns about a carious cavity in the 44tooth. Objectively: the 44 tooth has adeep carious cavity in the masticatorysurface, the dentine is dense and pi-gmented. Probing, percussion cause nopain. Electroodontodiagnosis is 12 mi-croampere. What is the most likely di-agnosis?

A. Chronic deep cariesB. Chronic median cariesC. Chronic fibrous periodontitisD. Chronic fibrous pulpitisE. Acute deep caries

140. A 32-year-old female patientcomplains about a cosmetic defect ofthe 14, 25 teeth. It is planned to makemetal-ceramic crowns. What impressionmaterial should be used for making thesecrowns?

A. SiliconeB. HardeningC. WaxD. AlginateE. Zinc oxide eugenol

141. A 25-year-old patient complainsabout a light brown spot in the upperforetooth. Objectively: the 23 tooth has asingle light brown spot in the cervical regi-on. Probing shows smooth surface. Thetooth is nonresponsive to cold and probi-ng. What is the most likely diagnosis?

Page 20: Krok 2 - 2010 Question Paper (Stomatology)

Krok 2 Stomatology 2010 20

A. Chronic initial cariesB. FluorosisC. Local enamel hypoplasiaD. Acute initial cariesE. Chronic superficial caries

142. A 60-year-old patient underwentssanitation of the oral cavity before anoperation on account of cataract. Afterexamination the patient was diagnosedwith chronic median caries of the 22 tooth(Black’s class V). What filling materialshould NOT be used in this patient?

A. Light-cure microhybrid materialB. Chemical-cure microhybrid materialC. Chemical-cure glass-ionomer cementD. Silicate cementE. Chemical-cure macrofilled composite

143. A 33-year-old patient complai-ns about an ulcer of oral cavity floor,that is located under his tongue on alevel between the 43 to the 33 tooth.Examination reveales that ulcer edgesare undermined and scalloped. Its grey-yellow floor is shallow and it is coveredwith small, easily bleeding granulations.There is no ulcer infiltration. Make a cli-nical diagnosis:

A. Tubercular ulcer of oral cavity floorB. Decubital ulcer of oral cavity floorC. Cancerous ulcer of oral cavity floorD. Gummatous ulcer of oral cavity floorE. Migratory granuloma of oral cavity floor

144. A steeplejack with a long record ofservice consults a dentist about dryness,burning and insignificant lip edema. Thesame symptoms were noted one year agoin autumn. What is the most likely di-agnosis?

A. Meteorogical cheilitisB. Contact cheilitisC. Cheilitis glandularisD. Manganotti’s cheilitisE. Cheilitis exfoliativa

145. A 42-year-old patient complainsabout gingival pain, progressing gingi-val haemorrhage, increasing tooth mobi-lity, halitosis. Objectively: gums are evi-dently hyperaemic, extremely edematic,they bleed easily on palpation. Tooth rootsare exposed, parodontal pouches are 4-6mm deep, and contain purulent exudate,there is also supragingival and subgingivaldental calculus. II-III grade tooth mobi-lity is present. Orthopantomogram showsresorption of interdental septa down to1/2 of their height. What is the most likely

diagnosis?

A. Exacerbation of generalized II degreeperiodontitisB. Exacerbation of generalized I degreeperiodontitisC. Exacerbation of generalized III degreeperiodontitisD. Chronic generalized II degree peri-odontitisE. Chronic generalized III degree peri-odontitis

146. A female patient applied to theoral surgery department and underwentradical maxillary sinusotomy with plasticrepair of fistula through the alveolarsocket of the extrated 27 tooth. Infiltrationand all the peripheral block anaesthesiasof the left upper jaw were performed wi-th 6,0 ml of 2% lidocaine solution. 3 mi-nutes later the patient registered doublevision in her left eye, inability to closeit. Which of the performed anaesthesi-as is the reason for the above-mentionedpresentations?

A. InfraorbitalB. TuberalC. PalatinalD. IncisorE. Infiltration

147. On the 7th day after the operation onthe abdominal cavity a 30-year-old pati-ent presented with pain and sweling in theregion of parotid gland; body temperaturerise up to 39oC; limited mouth opening,dryness. Gland massaging results in di-scharge of purulent exudate from its duct.The patient can be diagnosed with:

A. Acute non-epidemic parotiditisB. Acute epidemic parotiditisC. Phlegmon of submasseteric spaceD. Parenchymatous parotiditisE. Phlegmon of parotidomasseteric region

148. A 28-year-old patient complainsabout constant localised pain that is getti-ng worse when biting down on food.Objectively: the 46 tooth has a cariouscavity communicating with the dentalcavity. Probing of the carious cavity causesno pain, percussion is painful, X-ray pi-cture shows no changes. What is the mostlikely diagnosis?

Page 21: Krok 2 - 2010 Question Paper (Stomatology)

Krok 2 Stomatology 2010 21

A. Acute serous periodontitisB. Exacerbation of chronic pulpitisC. Acute purulent periodontitisD. Acute diffuse pulpitisE. Exacerbation of chronic periodontitis

149. A 23-year-old patient complainsabout a carious cavity in the 16 tooth.Objectively: the 16 tooth has a deep cari-ous cavity communicating with the dentalcavity. Probing, percussion cause no pain.There is a fistula on the gingiva in the regi-on of root apex projection of the 16 tooth.What is the most likely diagnosis?

A. Chronic granulating periodontitisB. Chronic gangrenous pulpitisC. Chronic fibrous periodontitisD. Chronic fibrous pulpitisE. Chronic granulomatous periodontitis

150. A 30-year-old male patient consulteda dentist about a swelling in the regionof his upper lip. Objectively: the face isasymmetric because of upper lip edema,nasolabial fold is smoothed. Mucogingivalfold of the upper jaw vestibular surfacein the region of the 11, 12, 21 teeth issmoothed, hyperemic. Palpation is pai-nful. Fluctuation is present. The patientwas diagnosed with acute purulent peri-ostitis of the upper jaw satrting from the21 tooth. Choose the treatment tactics:

A. Preservation of the 21 tooth, peri-osteotomy, anti-inflammatory therapyB. Preservation of the 21 tooth, anti-inflammatory therapyC. Extraction of the 21 tooth, peri-osteotomy, ant-inflammatory therapyD. Extraction of the 21 tooth, peri-osteotomyE. Extraction of the 12, 21, 22 teeth,periosteotomy, anti-inflammatory therapy

151. A 33-year-old officer deliveredfrom a battlefield has shoot off mentalpart o fmandible, drooling, insignifi-cant haemorrhage from the oral cavity,asphyxia that occurs periodically becauseof head repositioning. What actionsshould be taken to prevent the asphyxiadevelopment?

A. Fixation of tongue, transportation inprone positionB. Thacheostome constructionC. Fixation of jaws with a mental slingD. Surgical d-bridement, tracheostomeconstructionE. Transportation in prone position

152. During extraction of the 14 tooth

a 64-year-old patient complained aboutretrosternal pain irradiating to his back.There is paleness of membrane and skinintegument as well as pulse assymetry onthe carotid arteries. What is the most li-kely diagnosis?

A. Acute myocardial infarctionB. Hypertensic crisisC. Acute respiratory failureD. Angioneurotic Quincke’s edemaE. Collapse

153. Renovation of a dental office involvesinstallation of luminous tube lamps. Speci-fy the required illuminance level in lux:

A. 500 lxB. 400 lxC. 300 lxD. 200 lxE. 100 lx

154. A 42-year-old builder complainsabout a condyloma on his lower lip. Itappeared 1,5 month ago. It has been si-gnificantly growing throughout the lastweek. Objectively: the red border of thelower lip is cyanotic and infiltrated, it hassome isolated closly adhering squamae.There is a well-defined hemisphericalformation 8 mm in diameter and 4 mmhigh in the centre. The formation is ofgrey-and-blue-and-red colour, it has roughsurface formed by thin, closely adheri-ng and thick-based squamae. Regionallymph nodes are enlarged, mobile, denseand painless. What is the most likely di-agnosis?

A. Lower lip cancerB. Verruciform precancerC. Pyogenic granulomaD. KeratoacanthomaE. Viral wart

155. During the removal of dermoid cystof the nose bridge a 14-year-old boycomplained about giddiness, sickness,weakness. Objectively: the skin is pale,covered with cold sweat. Tachypnoe ispresent. The pulse is weak, the arterialpressure is low (80/60 mm Hg), hands feelcold. What is the most likely diagnosis?

A. SyncopeB. Traumatic shockC. Anaphylactic shockD. CollapseE. Toxic shock

156. A patient complains about pain in theoral cavity, burning and dryness. Exami-

Page 22: Krok 2 - 2010 Question Paper (Stomatology)

Krok 2 Stomatology 2010 22

nation revealed fiery-red dry mucousmembrane. The tongue is crimson, dry,glossy, filiform papillae are atrophied.There is some deposit in tongue foldsthat is hard to be removed. The patientundergoes treatment for pneumonia, shetakes antibiotics. What is the most likelydiagnosis?

A. Acute atrophic candidiasisB. Fastened erythemaC. Pellagrous glossitisD. B2 hypovitaminosisE. Benign migratory glossitis

157. A dentist applied to the regional sani-tation center for a permission to open aprivate dental office with two universaldental sets. Planned area of the recepti-on room is 26 �

2. According to existingnorms, the dental office with two universaldental sets must have the following area:

A. 14 �2 for each set and 10 �2 in additionB. 10 �2 for each set and 10 �2 in additionC. 10 �2 for each set and 7 �2 in additionD. 7 �2 for each set and 7 �2 in additionE. 20 �2 for each set and 12 �2 in addition

158. A 40-year-old patient underwentselective grinding of teeth on accountof TMJ disease provoked by functionalocclusal disorder. 3 days later the patientconsulted the orthopaedist about injuri-ng buccal mucosa because of its gettingin between dentitions on the right. Whatmistake was made during teeth grinding?

A. Tops of the support palatal tubercles ofthe upper teeth were ground offB. Tops of the buccal tubercles of the lowerteeth were ground offC. Lingual tubercles of the lower teethwere ground offD. Buccal tubercles of the upper teethwere ground offE. Upper teeth tubercles were ground off

159. A 20-year-old patient was diagnosedwith chronic deep caries of the 12 tooth.Objectively: the 12 tooth has a deep cari-ous cavity on the aproximal-distal surface.Distal crown angle of the 12 tooth is alsoaffected with caries. Choose the materialfor correction of the indicated defect:

A. Light-cure composite materialB. Chemical-cure glass ionomer cementC. Light-cure glass-ionomer cementD. Polycarboxylate cementE. Silica-alumina cement

160. A 9-year-old child complains about

dull pain that is getting worse while pressi-ng the tooth. The tooth was treated forpulpitis one mounth ago. Objectively: the36 tooth is filled. Percussion causes acutepain. Mucous membrane is hyperemicand edematic. What is the most likely di-agnosis?

A. Acute serous periodontitisB. Exacerbation of chronic pulpitisC. Acute diffuse pulpitisD. Acute suppurative pulpitisE. Exacerbation of chronic periodontitis

161. A 24-year-old victim consulted adoctor about a face burn. He was injuredby the open flame. He complains aboutpain and burning of face skin. Objecti-vely: hyperemia of face skin; in the regi-on of chin, nose, forehead, eyebrows,cheekbones there are burn blisters contai-ning transparent liquid. These presentati-ons correspond with the following degreeof burn severity:

A. I-II degreeB. II-III A degreeC. II-III B degreeD. III B-IV degreeE. II-IV degree

162. A 20-year-old patient complainsabout inability to move with his lowerjaw, speech difficulty and problems duri-ng eating. He associates his condition wi-th a trauma he got when biting on apple.Examination revealed half-open mouth,open bite due to the solitary contacts ofdistal tubercles of the last molars, salivati-on, indistinct speech. Articular heads ofmandible can be felt anterior to both tragi.What is the most likely diagnosis?

A. Acute anterior bilateral mandibleluxationB. Fibrous ankylosis of temporomandi-bular jointC. Exacerbation of osteoarthritis oftemporomandibular jointD. Traumatic bilateral fracture of articularprocesses of mandibleE. Fracture of articular processes of mandi-ble

163. A 6,5-year-old child has a gap 2,5-3 mm large between frontal teeth fromcanine to canine. Relationship of the firstpermanent molars complies with Angle’sclass I. Specify the severity degree of bitedeformation:

Page 23: Krok 2 - 2010 Question Paper (Stomatology)

Krok 2 Stomatology 2010 23

A. I degreeB. II degreeC. III degreeD. IV degreeE. V degree

164. A 5-year-old child complains aboutspontaneous pain in an upper jaw tooth onthe right that is getting worse at night andduring eating cold food. Objectively: the65 tooth has a deep cavity communicatingwith the tooth cavity. Probing is painful,percussion is painless. Cold water causeslong-standing pain. What is your provisi-onal diagnosis?

A. Exacerbation of chronic pulpitisB. Acute periodontitisC. Exacerbation of chronic periodontitisD. Acute serous pulpitisE. Acute purulent pulpitis

165. A 13-year-old boy complains aboutpain in the upper jaw caused by warmand hot stimuli, offensive breath when hesucks his tooth. Objectively: the 24 toothis changed in color, there is a deep cari-ous cavity communicating with the toothcavity. Deep probing is painful. Percussi-on causes no pain. What is the most likelydiagnosis?

A. Chronic gangrenous pulpitisB. Chronic periodontitisC. Chronic fibrous pulpitisD. Exacerbation of chronic pulpitisE. Exacerbation of chronic periodontitis

166. A 30-year-old patient complainsabout body temperature rise up to 39, 0oC,a roundish infiltrate on his upper lip,general weakness. He has been presentingwith these symptoms for 3 days. Objecti-vely: a roundish infiltrate in the regi-on of the upper lip 2,5 cm in diameter,the skin over the infiltrate is red with anecrotic core in the centre. The upper lipis hyperemic and edematic. What is themost likely diagnosis?

A. Furuncle of the upper lipB. Carbuncle of the upper lipC. Retention cyst of the upper lipD. Acute periostitis of the upper jawE. Acute glandular abscess

167. Parents of a 6-year-old child complainabout pain in the child’s submandibularregion on the left, body temperature ri-se up to 37, 5oC. Objectively: the child’sface is asymmetric due to the infiltrationof the submandibular region on the left.The infiltration is soft and elastic, mobile,

2х2,5 cm large; its palpation is slightly pai-nful, the skin is unchanged. The teeth areintact. Pharynx is hyperaemic. What is themost likely diagnosis?

A. Acute serous nonodontogenicsubmandibular lymphadenitisB. Acute serous odontogenic submandi-bular lymphadenitisC. Acute purulent nonodontogenicsubmandibular lymphadenitisD. Acute purulent odontogenic submandi-bular lymphadenitisE. Submandibular adenophlegmon

168. A 48-year-old woman complainsabout aching dull pain in the region ofthe left TJM, that is getting worse duri-ng eating solid food. The pain appearedabout 2,5 years ago. Objectively: mouthopening is limited, there is sideward devi-ation of jaw during mouth opening, TMJis clicking. Examination of the oral cavi-ty revealed secondary partial adentia. X-ray picture shows sclerosis of the corticalplate of articulat head and narrowing ofcartilage space. What is the most likely di-agnosis?

A. Arthrosis of the TMJB. Chronic arthritis of the TMJC. Acute arthritis of the TMJD. Painful dysfunction of the TMJE. Exacerbation of chronic arthritis of theTMJ

169. A 30-year-old patient was di-agnosed with posttraumatic osteomyeli-tis of mandible in the region of mandi-ble body on the right. The patient gotthe trauma 1,5 month ago. X-ray pictureshows the fracture line and the shadow ofosseous sequestre along the fracture line.There is a fistula with purulent dischargeon skin. Choose the treatment tactics:

A. Fistulectomy, sequestrectomyB. PhysiotherapyC. SequestrectomyD. FistulectomyE. Fistulectomy, antibacterial therapy

170. A dentist fills a Black class II cari-ous cavity in the 36 tooth of a 35-year-oldman by sandwich method in one office vi-sit. What glass-ionomer cement should bechosen as basic liner in this case?

Page 24: Krok 2 - 2010 Question Paper (Stomatology)

Krok 2 Stomatology 2010 24

A. HybridB. Water-hardeningC. ClassicalD. CondensableE. Reinforced

171. A 54-year-old male patient complai-ns about progressing pain in the area ofupper jaw on the left that first arose 3 daysago. Body temperature is up to 37, 2oC.Objectively: gingival mucous membraneis edematic, hyperemic, it bleeds slightly.Parodontal pouches are 6-7 mm deepand contain a small amount of purulentexudate. A painful infiltrate is palpated inthe region of the 26 tooth. The 26 toothhas III degree mobility, the rest of teethhave I-II degree mobility. X-ray pictureshows resorption of interalveolar septaby 2/3 of root length. What is the mostappropriate treatment method of the 26tooth?

A. Extraction of the 26 toothB. Temporal splintage, conservativetherapyC. Curettage of parodontal poucheD. Lancing of abscess via gumE. Lancing of abscess via parodontal pouch

172. A 27-year-old victim was delivered toto the facial surgery department with acut wound of right cheek. The trauma iscivilian, the patient got it in a fight. Thedeadline for early surgical d-bridementwithout preliminary antibiotic injection isno more than:

A. 24 hoursB. 48 hoursC. 72 hoursD. 12 hoursE. 1 hour

173. A 62-year-old patient has a medianfracture of mandible along with formati-on of a false joint. Objectively: dentalformula is 33, 34, 35, 36, 27, 47, 46, 45, 44,43. The teeth are intact, stable, with highcrowns. Fragment mobility is insignificant,there is no displacement. X-ray pictureshows a bone defect 0,8 cm large. Whatprosthesis is indicated?

A. Bridge-like prosthesis with a pivot pointB. Lamellar prosthesis without a pivotpointC. Lamellar prosthesis with Gavrilow’spivot pointD. Lamellar prosthesis with Oxman’s pivotpointE. Lamellar prosthesis with Weinstein’spivot point

174. After supercooling a 42-year-old pati-ent presented with headache in the leftfrontal region and left upper jaw. Objecti-vely: the face is symmetric, breathi-ng through the left nasal meatus isobstructed, seropurulent discharges arepresent. Palpation is slightly painful inthe infraorbital region as well as alongthe mucogingival fold in projection of the24, 25 teeth. Percussion of these teeth ispainless. The 24 tooth is filled. Mucuousmembrane of alveolar process has no visi-ble changes. X-ray picture shows reducedpneumatization of the left upper jaw si-nus. What is the provisional diagnosis?

A. Exacerbation of chronic odontogenicmaxillary sinusitisB. Acute periodontitis of the 24 toothC. Exacerbation of chronic periodontitis ofthe 24 toothD. Acute rhinogenous maxillitisE. Acute albuminous periostitis of the leftupper jaw

175. An 8,5-year-old child is apparentlyhealthy. The child complains about painin an upper tooth on the left caused bytraumatic injury sustained three hoursago. Objectively: the crown part of the 21tooth is destroyed by 1/2, the pulp is redand significantly exposed, probing causesacute pain and bleeding. Percussion of the21 tooth is extremely painful. Choose themost efficient treatment method of the 21tooth:

A. Vital amputationB. Vital extirpationC. Devital amputationD. Devital extirpationE. Bioassay technique

176. A 36-year-old female patientcomplains about an aesthetic defectof upper frontal teeth. In past hi-story: she undergoes regular check-upin the endocrinological department onaccount of thyrotoxicosis. Enamel defectsappeared several years ago and since thathave been gradually expanding. Exami-nation revealed that the 12, 11, 21, 22

Page 25: Krok 2 - 2010 Question Paper (Stomatology)

Krok 2 Stomatology 2010 25

teeth had transverse oval enamel defectson the vestibular surfaces. Probing andcold test is painless. Floor of the defects issmooth, glossy and hard. What is the mostlikely diagnosis?

A. Erosion of hard tissiesB. Acute superficial cariesC. Wedge-shaped defectsD. Systemic hypoplasiaE. Necrosis of hard tissues

177. A 23-year-old patient complai-ns about root exposition, gingivalhemorrhage during tooth brushing, gumitch. Objectively: there is supragingivaland subgingival dental calculus. Gums arehyperaemic, edematic, pouches are 3,5mm deep. X-ray picture shows resorpti-on of interalveolar septa by 1/3. What isthe most likely diagnosis?

A. Chronic generalized I degree peri-odontitisB. Chronic generalized II degree peri-odontitisC. Exacerbation of generalized I degreeperiodontitisD. Exacerbation of generalized II degreeperiodontitisE. II degree periodontitis

178. A 14-year-old child complains aboutacute spontaneous pain in an upper jawtooth on the right. The pain has beenlasting for 3 days, it is throbbing, irradi-ating to the temple, getting worse at ni-ght. Objectively: surface of the 15 toothexhibits a carious cavity within parapulpardentine. Dentine is softened, of greyishcolour. Probing of the whole cavity floor ispainful, percussion of the 15 tooth is pai-nless. What is the most likely diagnosis?

A. Acute purulent pulpitisB. Acute diffuse pulpitisC. Acute focal pulpitisD. Acute periodontitisE. Exacerbation of chronic periodontitis

179. A child’s mother had acute purulentmastitis. Now the 2-month-old childexperiences an edema of his left infraorbi-tal and malar regions, dermahemia ofthe left face side, temperature rise up to39 − 40oC, purulent discharges from thenose. What is the most likely diagnosis?

A. Acute hematogenous osteomyelitisB. Phlegmon of infraorbital regionC. Acute maxillary sinusitisD. Acute odontogenic osteomyelitisE. Acute purulent periostitis

180. According to the parents of a 7-year-old child, the child complains aboutweakness, body temperature rise up to39oC, toothache in the upper jaw onthe left. Objectively: condition is grave,the child is pale and adynamic, the faceis asymmetric because of infiltration ofthe upper jaw on the left. The 64 toothhas a carious cavity. Percussion is pai-nful, I degree of teeth mobility is alsopresent. There are purulent dischargesfrom the subgingival edge of the 64tooth. Mucogingival fold of the 63, 64, 65teeth is smoothed. Vestibular and palatinemucous membrane is edematic. What isthe most likely diagnosis?

A. Acute odontogenic maxillaryosteomyelitis starting from the 64 toothB. Acute albuminous maxillary periostitisC. Acute purulent odontogenic maxillaryperiostitis starting from the 64 toothD. Suppurative radicular cyst of maxillaE. Acute odontogenic maxillary sinusitis

181. A 35-year-old man has a medi-um deep carious cavity in the 37 tooth(Black’s class II). For its filling a denti-st chose technique of layer-by-layerrestoration. What composite should becoating the floor and walls of the cariouscavity in order to form superadaptive ini-tial layer?

A. FlowableB. CondensableC. MacrofilledD. MicrohybridE. Microfilled

182. A 32-year-old male patient was deli-vered to traumatology. He complainsabout nosebleed, severe cephalalgia, gi-ddiness, face deformation, heavy breathi-ng, mastication inability. He sustainedan injury in the mine over three hoursago. He was conscious. Objectively: themiddle third of his face is elongated,there are considerable hematomas andthere is an edema in the region of noseroot and eyelids of both eyes. The upperjaw is displaced downwards, it is mobiletogether with nasal bones, cheek bonesand eyeballs. Palpation reveals deformati-on, pain and pathological mobility in theregion of the nose bridge, external socket

Page 26: Krok 2 - 2010 Question Paper (Stomatology)

Krok 2 Stomatology 2010 26

edges, pterygoid processes of basilar bone.Nosebleed is present. What is the most li-kely diagnosis?

A. Le Fort III maxillary fracture (upper)B. Le Fort I maxillary fracture (lower)C. Le Fort II maxillary fractureD. Fracture of zygomatic boneE. Fracture of nasal bone

183. A 9-year-old boy presents withface asymmetry due to the chin devi-ation to the left. When the third Il’ina-Marcosian diagnostic test is performedface asymmetry disappears. What is themost likely clinical form of this occlusalabnormality?

A. Habitual deviation of mandibleB. Ankylosis of the temporomandibularjointC. Unilateral hypoplasia of mandibleD. Bilateral narrowing of the maxillarydental archE. Unilateral narrowing of the maxillarydental arch

184. A 13-year-old girl has been experi-encing gingival hemorrhages and frontalteeth mobility throughout the last month.Objectively: gingival mucous membranein the region of lower incisors and caninesis edematic, hyperemic, it bleeds on touch.These teeth present with I degree mobili-ty, parodontal pouches are 3 mm deep.Orthopantomogram shows reduction ofbone tissue of interalveolar septa by 1/3of their height. Frontal teeth of the lowerjaw are overcrowded. Hygiene index is4,2. What local treatment should be provi-ded in the first place?

A. Professional hygieneB. Anti-inflammatory therapyC. Orthodontic careD. Physical therapyE. Antibacterial therapy

185. A 16-year-old teenager complainsabout halitosis, general weakness, bodytemperature rise up to 37, 6oC. Thesesymptoms turned up two days ago, the boyhad a history of recent angina. Objecti-vely: oral hygiene is unsatisfactory, theteeth are coated with soft dental deposit.Gums are hyperemic, gingival papillae arecovered with greyish deposit. Specify thecausative agent of this disease:

A. Fusobacteria, spirochetesB. StreptococciC. Herpes virusesD. StaphylococciE. Yeast fungi

186. A 65-year-old male patient complai-ns about crepitation and clicking in bothtemporomandibular joints, pain inducedby displacement of the lower jaw to theright, ear noise, dry mouth, glossalgia.He has been using complete removableprosthesis of the lower jaw for 6 months.The patient denies rheumatosis. Objecti-vely: the lower third of face is shortened,mental fold is strongly pronounced,mouth corners are lowered, angular fi-ssures and cracks are also present. Palpati-on reveals crepitatnt rale observed duringTMJ moving. What is the most likely di-agnosis?

A. Costen’s syndromeB. Temporomandibular arthritisC. Temporomandibular arthrosisD. Temporomandibular dislocationE. Temporomandibular osteoarthritis

187. A 40-year-old patient complainsabout a long-existing fissure in the centralpart of red border of his lower lip. He hasbeen smoking since he was 16. Objecti-vely: there is a deep 1,0 cm long fissurealong the median line of red border ofthe lower lip, the fissure edges are bulgi-ng. Characteristic quality of the fissureis its spontaneous healing, but after epi-thelization the fissure recurs. The pati-ent was diagnosed with chronic fissureof the lower lip. Conservative treatmentproved to be ineffective. Choose the mostappropriate treatment method in this si-tuation:

A. Excision within evidently healthy ti-ssuesB. CryodestructionC. Diathermo-coagulationD. Excision 0,5 cm off the neoplasm limitsE. Close-focus roentgenotherapy

188. A 52-year-old patient complai-ns about teeth mobility, gingivalhaemorrhage. The 44, 45, 34, 35 teethpresent with I degree mobility, the 43,42, 41, 31, 32, 33 teeth present with IIdegree of teeth mobility. There is gingivaledema, parodontal pouches are 5-6 mmdeep. X-ray picture shows destruction ofbone tissue by 1/2 of root length. A dentistmade a diagnosis: generalized periodonti-tis of median severity. What orthopaedic

Page 27: Krok 2 - 2010 Question Paper (Stomatology)

Krok 2 Stomatology 2010 27

construction is the most efficient in thissituation?

A. Elbrecht splintB. Plastic mouthguardC. Murray’s splintD. Mamlock splintE. Temporary Novotny splint

189. A victim got a face and temporal regi-on trauma. A doctor made a diagnosis:fracture of malar arch. What processes ofcranial bones are injured?

A. Temporal process of zygomatic boneand zygomatic process of temporal boneB. Zygomatic process of frontal bone andzygomatic process of temporal boneC. Temporal process of zygomatic boneand zygomatic process of frontal boneD. Zygomatic process of maxilla andzygomatic process of temporal boneE. Zygomatic process of maxilla andzygomatic process of frontal bone

190. A 28-year-old patient had toundergo extraction of the 46 toothunder Weisbrem’s apodactylic mandibularanaesthesia. What is the target site of theanaesthetic solution injection?

A. A segment of mandibular torusB. A segment of maxillary tuberC. Retromolar fossaD. A segment of temporal crest of mandi-bleE. Retromandibular fossa

191. A 7-year-old child sustained a sportinjury. He complains about pain in theregion of mandible, inability to join histeeth. Pressing against the chin causesintensification of pain in the region ofmandible on the right. The child is di-agnosed with a fracture of mandible bodywithout fragment displacement. Whattherapeutic tactics should be chosen inorder to prevent the displacement of bonefragments in the transitional dentition?

A. Fragment immobilization by means ofan intraoral splintB. Administration of anti-inflammatorytherapyC. Application of sling bandageD. Fixation by means of Tigerstedt splintsE. Osteosynthesis

192. A 44-year-old patient complainsabout the astringent sensation in theupper jaw incisors, which appeared 1,5years ago. Objectively: the most convexpart of the vestibular surfaces of the 12,

11, 21, 22 crowns has roundish defects wi-th smooth, shiny, dense surface, that reachdentine in depth. The depth of defects isgradually decreasing from the centre tothe periphery. What is the most likely di-agnosis?

A. Enamel erosionB. Wedge-shaped defectC. Systemic hypoplasiaD. Chronic median cariesE. Destructive fluorosis

193. During the planned oral cavity sani-tation an 11,5-year-old girl complainedabout periodic spontaneous chilalgiaduring food intake. Objectively: mucousmembrane of lips is hyperemic, edematicand dry. During talking one can observesmall drops of mucous secretion. Makethe provisionaldiagnosis:

A. Cheilitis glandularisB. Angular cheilitisC. Eczematous cheilitisD. Contact allergic cheilitisE. Cheilitis exfoliativa

194. A 38-year-old patient consulted adentist about aching pain in the region ofthe 48 tooth that is getting worse when bi-ting down on food. Body temperature riseup to 37, 6oC and aggravation of generalcondition is present. Objectively: insignifi-cant swelling of retromolar region as wellas hyperaemia of mucous membrane. The48 tooth is covered with occlusal pad. It ispainful on palpation. Purulent dischargesare present. What is the most likely di-agnosis?

A. Acute purulent pericoronitisB. Acute pulpitisC. Acute periodontitis of the 48 toothD. Aphthous stomatitisE. Acute purulent periostitis

195. A patient ordered metal-ceramiccrowns for the 11, 12 teeth. Blockanaesthesia was performed beforepreparation. In a few minutes the pati-ent felt anxiety, tinnitus, abdominal pain,giddiness. Breathing is hurried, pulse israpid, AP is 60/40 mm Hg. What aidshould be rendered to the patient in thefirst place?

Page 28: Krok 2 - 2010 Question Paper (Stomatology)

Krok 2 Stomatology 2010 28

A. Intravenous introduction of 0,5 ml of0,1% adrenaline solution, prednisoloneB. Promedol, suprastinC. To soak a cotton web with some liquidammonia and give it for the patient toinhaleD. To bring the patient into horizontalpositionE. Make an injection of aminophylline

196. A 40-year-old female patientcomplains about crepitation and pain inthe region of TMJ projection on the ri-ght, that arise in the final stage of mouthopening. Articular sounds turned up afterdental prosthetics. Mouth opening is unli-mited, mouth is opened in a straight path,amplitude of mouth opening is 5 cm. Whatis the most likely diagnosis?

A. Lower jaw subluxationB. Lower jaw dislocationC. Anterior dislocation of articular diskD. Posterior dislocation of articular diskE. Meniscus tresis

197. A 73-year-old patient consults adental orthopaedist about total edentia.Objectively: there is insignificant regularatrophy of the alveolar process ofmandible, the process is covered withmucous membrane of moderate elasticity.According to Keller’s classification, suchmanifestations are characteristic for thefolowing type of edentulous jaw:

A. FirstB. ThirdC. FifthD. SecondE. Fourth

198. An 18-year-old patient complai-ns about ulceration in the oral cavi-ty, spontaneous bleeding of mucousmembrane, pain during food intake andtalking, nosebleeds. He has a historyof: aggravation of general condition,

weakness, body temperature rise up to39oC, headache, joint pain. What methodof diagnostics should be applied to confi-rm the diagnosis?

A. Clinical blood analysisB. Blood sugar testC. ImmunogramD. HIV testE. Allergy test

199. A 14-year-old girl complains about anaesthetic defect in form of white spots onher teeth. Fluorine concentration in thedrinking water in the area of her residenceis 2 mg/l. Objectively: vestibular surfacesof all teeth are covered with ill-definedwhite spots. What is the most likely di-agnosis?

A. FluorosisB. Local hypoplasiaC. Systemic hypoplasiaD. Multiple cariesE. Amelogenesis imperfecta

200. A 42-year-old patient consulteda dentist about intense lancinatingparoxysmal pain accompanied by asensation of current passage in the regi-on of her upper lip on the right. Painattacks occur spontaneously and last 3-5 minutes. The patient usually has 2-3attacks a day. The patient is unable toestablish the cause of this disease. Exami-nation of her oral cavity revealed nopathological changes. Oral cavity is sani-tated. Test orthopantomogram shows animpacted supernumerary 13 tooth. Whatis the most likely diagnosis?

A. Peripheral neuralgia of the II branch oftrigeminusB. Pterygopalatine ganglionitisC. Neuritis of the II branch of trigeminusD. Central neuralgia of the II branch oftrigeminusE. Right-sided upper jaw pulpitis